Pulmonology Last

Download as doc, pdf, or txt
Download as doc, pdf, or txt
You are on page 1of 82

PULMONOLOGY

Question 1
Which of the following factors is least useful in assessing patients with a poor prognosis in community-acquired
pneumonia?
A. Mini-mental score of 6/10
B. Urea of 11.4 mmol/l
C. C-reactive protein of 154
D. Respiratory rate of 30
E. Aged 75 years old

The C-reactive protein is the least useful of the above in predicting mortality in patients with community-
acquired pneumonia. The rest of the answers are part of the CURB-65 criteria
Pneumonia: prognostic factors
CURB-65 criteria of severe pneumonia
Confusion (abbreviated mental test score < 8/10)
Urea > 7 mmol/L
Respiratory rate >= 30 / min
BP: systolic < 90 or diastolic < 60 mmHg
age > 65 years
Patients with 3 or more (out of 5) of the above criteria are regarded as having a severe pneumonia
Other factors associated with a poor prognosis include:
presence of coexisting disease
hypoxaemia (pO2 < 8 kPa) independent of FiO2

Question 2
A 24-year-old female presents with episodic wheezing and shortness of breath for the past 4 months. She has
smoked for the past 8 years. Examination of her chest is unremarkable. What is the most appropriate
management of her symptoms?
A. Peak flow diary
B. Spirometry
C. Baseline FEV1 repeated following inhaled corticosteroids
D. Baseline FEV1 repeated following inhaled salbutamol
E. Trial of salbutamol inhaler

Asthma diagnosis - if high probability of asthma - start treatment

The new British Thoracic Society guidelines take a more practical approach to
diagnosing asthma. If a patient has typical symptoms of asthma a trial of
treatment is recommended. The smoking history is unlikely to be relevant at her
age

Asthma: diagnosis in adults


sqweqwesf erwrewfsdfs adasd dhe
The 2008 British Thoracic Society guidelines marked a subtle change in the
approach to diagnosing asthma. It suggests dividing patients into a high,
intermediate and low probability of having asthma based on the presence or
absence of typical symptoms. A list can be found in the external link but
include typical symptoms such as wheeze, nocturnal cough etc

Example of features used to assess asthma (not complete, please see link)

Increase possibility of asthmaDecrease possibility of asthma


• Wheeze, breathlessness, chest tightness and cough, worse at night/early
morning
• History of atopic disorder
• Wheeze heard on auscultation
• Unexplained peripheral blood eosinophilia• Prominent dizziness,
light-headedness, peripheral tingling
• Chronic productive cough in the absence of wheeze or breathlessness
• Repeatedly normal physical examination
• Significant smoking history (i.e. > 20 pack-years)
• Normal PEF or spirometry when symptomatic

Management is based on this assessment:

high probability: trial of treatment


intermediate probability: see below
low probability: investigate/treat other condition

For patients with an intermediate probability of asthma further investigations


are suggested. The guidelines state that spirometry is the preferred initial
test:

FEV1/FVC < 0.7: trial of treatment


FEV1/FVC > 0.7: further investigation/consider referral

Recent studies have shown the limited value of other 'objective' tests. It is
now recognised that in patients with normal or near-normal pre-treatment lung
function there is little room for measurable improvement in FEV1 or peak flow.

A > 400 ml improvement in FEV1 is considered significant

before and after 400 mcg inhaled salbutamol in patients with diagnostic
uncertainty and airflow obstruction present at the time of assessment
if there is an incomplete response to inhaled salbutamol, after either inhaled
corticosteroids (200 mcg twice daily beclometasone equivalent for 6-8 weeks)
or oral prednisolone (30 mg once daily for 14 days)

It is now advised to interpret peak flow variability with caution due to the
poor sensitivity of the test

diurnal variation % = [(Highest – Lowest PEFR) / Highest PEFR] x 100


assessment should be made over 2 weeks
greater than 20% diurnal variation is considered significant

Question stats

A23.3%
B12.9%
C3.4%
D24.5%
E35.8%

35.8% of users answered this question correctly


External links

British Thoracic Society


2008 Asthma guidelines

All contents of this site are ©2008 passmedicine.com - Terms and Conditions
passmedicine.compassmedicine.comReference ranges End session

Question 14 of 1500
Which one of the following is a contraindication to surgical resection in lung
cancer?ia A.AHaemoptysisia
B.AFEV 1.9 litresia
C.AHistology shows squamous cell canceria
D.AVocal cord paralysisia
E.ACalcium = 2.84 mmol/Lia

Contraindications to lung cancer surgery include SVC obstruction, FEV <


1.5, MALIGNANT pleural effusion, and vocal cord paralysis

Paralysis of a vocal cord implies extracapsular spread to mediastinal nodes and


is an indication of inoperability.

Lung cancer: non-small cell management


sqweqwesf erwrewfsdfs adasd dhe
Management

only 20% suitable for surgery


mediastinoscopy performed prior to surgery as CT does not always show
mediastinal lymph node involvement
curative or palliative radiotherapy
poor response to chemotherapy

Surgery contraindications

assess general health


stage IIIb or IV (i.e. metastases present)
FEV1 < 1.5 litres is considered a general cut-off point* ‫اطلع عكريتيريا المحاضرة‬
‫اسهل في الحفظ‬
malignant pleural effusion
tumour near hilum
vocal cord paralysis
SVC obstruction

* However if FEV1 < 1.5 for lobectomy or < 2.0 for pneumonectomy then some
authorities advocate further lung function tests as operations may still go
ahead based on the results

Question stats

A2.1%
B20%
C8.5%
D62.2%
E7.2%

62.2% of users answered this question correctly


External links

British Thoracic Society


BTS guidelines on selection for surgery

SIGN
Lung cancer management guidelines

All contents of this site are ©2008 passmedicine.com - Terms and Conditions
passmedicine.compassmedicine.comReference ranges End session

Question 46 of 1500
Which one of the following is responsible for farmer's lung?ia
A.AAspergillus clavatusia
B.AMicropolyspora faeniia
C.AThermoActinomyces candidusia
D.AMycobacterium aviumia
E.AAvian proteinsia

Micropolyspora faeni causes farmer's lung, a type of EAA

Extrinsic allergic alveolitis


sqweqwesf erwrewfsdfs adasd dhe
Extrinsic allergic alveolitis (EAA) is a condition caused by hypersensitivity
induced lung damage due to a variety of inhaled organic particles. It is thought
to be largely caused by immune-complex mediated tissue damage (type III
hypersensitivity) although delayed hypersensitivity (type IV) is also thought to
play a role in EAA, especially in the chronic phase

Examples

bird fanciers' lung (avian proteins)


farmers lung (spores of Micropolyspora faeni)
malt workers' lung (Aspergillus clavatus)
mushroom workers' lung (thermophilic actinomycetes*)

Presentation

acute: occur 4-8 hrs after exposure, SOB, dry cough, fever
chronic

Investigation

CXR: upper lobe fibrosis


BAL: lymphocytosis
blood: NO eosinophilia
*here the terminology is slightly confusing as thermophilic actinomycetes is an
umbrella term covering strains such as Micropolyspora faeni

Question stats

A18.5%
B57.4%
C14.9%
D5%
E4.2%

57.4% of users answered this question correctly


All contents of this site are ©2008 passmedicine.com - Terms and Conditions
passmedicine.compassmedicine.comReference ranges End session

Question 59 of 1500
Which one of the following causes of lung fibrosis predominately affect the
upper zones?ia A.ABleomycinia
B.ARheumatoid arthritisia
C.ACryptogenic fibrosis alveolitisia
D.AMethotrexateia
E.AExtrinsic allergic alveolitisia

Lung fibrosis
sqweqwesf erwrewfsdfs adasd dhe
It is important in the exam to be able to differentiate between conditions causing predominately upper or lower
zone fibrosis. It should be noted that the
more common causes (cryptogenic fibrosing alveolitis, drugs) tend to affect the
lower zones

Fibrosis predominately affecting the upper zones

extrinsic allergic alveolitis


coal worker's pneumoconiosis/progressive massive fibrosis
silicosis
sarcoidosis
ankylosing spondylitis (rare)
histiocytosis
tuberculosis

Fibrosis predominately affecting the lower zones

1 cryptogenic fibrosing alveolitis IPF


2 most connective tissue disorders (except ankylosing spondylitis)
3 drug-induced: amiodarone, bleomycin, methotrexate BBCMAN
4 asbestosis

Question stats

A10.3%
B9.7%
C10.8%
D7.6%
E61.6%

61.6% of users answered this question correctly


All contents of this site are ©2008 passmedicine.com - Terms and Conditions
passmedicine.compassmedicine.comReference ranges End session

Question 63 of 1500
A 41-year-old female presents with 3 day history of a dry cough and shortness of
breath. This was preceded by flu-like symptoms. On examination there is a
symmetrical, erythematous rash with 'target' lesions over the whole body. What
is the likely organism causing the symptoms?ia A.APseudomonasia
B.AStaphylococcus aureusia
C.AMycoplasma pneumoniaeia
D.AChlamydia pneumoniaeia
E.ALegionella pneumophiliaia

Pneumococcus may also cause erythema multiforme

Mycoplasma pneumoniae
sqweqwesf erwrewfsdfs adasd dhe
Mycoplasma pneumoniae is a cause of atypical pneumonia which often affects
younger patients. It is associated with a number of characteristic complications
such as erythema multiforme and cold autoimmune haemolytic anaemia. Epidemics of
Mycoplasma pneumoniae classically occur every 4 years. It is important to
recognise atypical pneumonias as they may not respond to penicillins or
cephalosporins

Features

flu-like symptoms classically precede a dry cough


bilateral consolidation on x-ray
complications may occur as below

Complications What is the extrapulmonary manifestation of mycoplasma ‫سؤال محمود شيخ علي‬
‫ بتقسمهم‬CNS, ear. Cardio, git, renal, skin, blood
cold agglutins (IgM) may cause an haemolytic anaemia, thrombocytopenia
erythema multiforme, erythema nodosum
meningoencephalitis, Guillain-Barre syndrome
bullous myringitis: painful vesicles on the tympanic membrane
pericarditis/myocarditis
gastrointestinal: hepatitis, pancreatitis
renal: acute glomerulonephritis

Diagnosis

Mycoplasma serology IgM

Management

erythromycin/clarithromycin

Question stats

A1.5%
B13.8%
C68.2%
D5%
E11.6%

68.2% of users answered this question correctly

September 2008 exam


All contents of this site are ©2008 passmedicine.com - Terms and Conditions
passmedicine.compassmedicine.comReference ranges End session

Question 101 of 1500


A 46-year-old female with a history of rheumatoid arthritis is investigated due
to progressive shortness of breath. She is currently treated with methotrexate
and ibuprofen. The following results are obtained from spirometry:

FEV1/FVC45%

What is the most likely cause of the dyspnoea?ia A.ABronchiolitis


obliteransia
B.AMethotrexate pneumonitisia
C.APulmonary fibrosisia
D.ACaplan's syndromeia
E.ALung canceria

The spirometry reveals an obstructive picture which would be in keeping with


bronchiolitis obliterans ,,,,mean give obstructive pattern

Rheumatoid arthritis: pulmonary manifestations


sqweqwesf erwrewfsdfs adasd dhe
A variety of respiratory problems may be seen in patients with rheumatoid
arthritis:

pulmonary fibrosis
pleural effusion
pulmonary nodules
bronchiolitis obliterans
complications of drug therapy e.g. methotrexate pneumonitis
pleurisy
Caplan's syndrome - massive fibrotic nodules with occupational coal dust
exposure
infection (possibly atypical) secondary to immunosuppression

Question stats

A47%
B18.1%
C25.9%
D8%
E1%

47% of users answered this question correctly

May 2006 exam


All contents of this site are ©2008 passmedicine.com - Terms and Conditions
passmedicine.compassmedicine.comReference ranges End session

Question 112 of 1500


You are reviewing the results from investigations requested at the previous
respiratory clinic. A 40-year-old man is being investigated for increasing
shortness of breath. The notes show he has smoked for the past 25 years.
Pulmonary function tests reveal the following:

FEV11.4 L
FVC1.7 L
FEV1/FVC82%

Which one of the following is the most likely explanation?ia A.AAsthmaia


B.ABronchiectasisia
C.AKyphoscoliosisia
D.AChronic obstructive pulmonary diseaseia
E.ALaryngeal malignancyia

These results show a restrictive picture, which may result from a number of
conditions including kyphoscoliosis. The other answers cause an obstructive
picture.

Pulmonary function tests


sqweqwesf erwrewfsdfs adasd dhe
Pulmonary function tests can be used to determine whether a respiratory disease
is obstructive or restrictive. The table below summarises the main findings and
gives some example conditions:

Obstructive lung diseaseRestrictive lung disease


FEV1 - significantly reduced
FVC - reduced or normal
FEV1% (FEV1/FVC) - reducedFEV1 - reduced
FVC - significantly reduced
FEV1% (FEV1/FVC) - normal or increased
Asthma
COPD
BronchiectasisPulmonary fibrosis
Asbestosis
Sarcoidosis
Acute respiratory distress syndrome
Infant respiratory distress syndrome
Bronchiolitis obliterans
Kyphoscoliosis
Neuromuscular disorders

Obstructive pattern:
Asthma , COPD, Bronchillitis obliterans, bronchiectasis obstructive cancer.

Question stats

A7.1%
B11.9%
C39.6%
D32.3%
E9.1%

39.6% of users answered this question correctly

September 2008 exam


All contents of this site are ©2008 passmedicine.com - Terms and Conditions
passmedicine.compassmedicine.comReference ranges End session

Question 121 of 1500


A 74-year-old woman with thyroid cancer is admitted due to shortness of breath.
What is the best investigation to assess for possible compression of the upper
passmedicine.compassmedicine.comReference ranges End session

Question 1081 of 1500


A 19-year-old male with no past medical history presents to the Emergency
Department with anterior chest pain and shortness of breath. Blood pressure is
110/80 mmHg and his pulse is 84 bpm. The chest x-ray is reported as showing a
50% pneumothorax with no mid-line shift. What is the most appropriate
management?ia A.AIntercostal drain insertionia
B.AImmediate 14G cannula into 2nd intercostal space, mid-clavicular
lineia
C.ADischargeia
D.AAdmit for 48 hours observation and repeat chest x-rayia
E.AAspirationia

Pneumothorax
sqweqwesf erwrewfsdfs adasd dhe
The British Thoracic Society (BTS) published guidelines for the management of
spontaneous pneumothorax in 2003. A pneumothorax is termed primary if there is
no underlying lung disease and secondary if there is

Primary pneumothorax

Recommendations include:

if the rim of air is < 2cm and the patient is not short of breath then
discharge should be considered
otherwise aspiration should be attempted
if this fails then repeat aspiration should be considered
if this fails then a chest drain should be inserted

Secondary pneumothorax

Recommendations include:

if the patient is > 50 years old and the rim of air is > 2cm and the patient
is short of breath then a chest drain should be inserted.
otherwise aspiration should be attempted. If aspiration fails a chest drain
should be inserted. All patients should be admitted for at least 24 hours

Iatrogenic pneumothorax

Recommendations include:

less likelihood of recurrence than spontaneous pneumothorax


majority will resolve with observation, if treatment is required then
aspiration should be used
ventilated patients need chest drains, as may some patients with COPD

Question stats

A23.1%
B10.4%
C2.6%
D2.8%
E61.1%
61.1% of users answered this question correctly
External links

British Thoracic Society


Pneumothorax guidelines

All contents of this site are ©2008 passmedicine.com - Terms and Conditions
B.AForced vital capacityia
C.ATransfer factoria
D.APeak expiratory flow rateia
E.AFlow volume loopia

Flow volume loop is the investigation of choice for upper airway


compression

Flow volume loop


sqweqwesf erwrewfsdfs adasd dhe
A normal flow volume loop is often described as a 'triangle on top of a semi
circle'

Flow volume loops are the most suitable way of assessing compression of the
upper airway

Question stats

A2.7%
B9.7%
C1.8%
D13.2%
E72.6%

72.6% of users answered this question correctly


All contents of this site are ©2008 passmedicine.com - Terms and Conditions
passmedicine.compassmedicine.comReference ranges End session

Question 160 of 1500


A 45-year-old female develops pleuritic chest pain following a hysterectomy 10
days ago. Low-molecular weight heparin is given initially and CTPA confirms a
pulmonary embolism. There is no previous history of venous thromboembolism. How
long should the patient be
warfarinised for?ia A.ANot suitable for anticoagulationia
B.A6 weeksia
C.A6 monthsia
D.A12 monthsia
E.ALife-longia

As this patient has a temporary risk factor for a thromboembolic event the
recommended period of anticoagulation is 4-6 weeks, according to the 2003
British Thoracic Society guidelines. Recent trial data may however change this
recommendation (see below)

Pulmonary embolism: management


sqweqwesf erwrewfsdfs adasd dhe
The British Thoracic Society (BTS) published guidelines in 2003 on the
management of patients with suspected pulmonary embolism (PE)

Key points from the guidelines include:

Anticoagulation

low molecular weight heparin (LMWH), rather than unfractionated heparin (UFH),
should be used routinely in patients with suspected pulmonary embolism. This
reflects the equal efficacy and safety of LMWHs as well as their ease of use.
Exceptions include patients with a massive PE or in situations where rapid
reversal of anticoagulation may be necessary
warfarinisation: the standard duration of anticoagulation is 4-6 weeks* if
temporary risk factors are present, 3 months for the first idiopathic PE and
at least 6 months for other situations

Thrombolysis

thrombolysis is now recommended as the first-line treatment for massive PE


where there is circulatory failure (e.g. hypotension). Other invasive
approaches should be considered where appropriate facilities exist

*the 2005 British Committee for Standards in Haematology guidelines suggest at


least 3 months anticoagulation in this situation, based on a 2004 study. It is
therefore possible these guidelines may soon change

Question stats

A2.5%
B45.5%
C47.8%
D2%
E2.3%

45.5% of users answered this question correctly


External links

British Thoracic Society


2003 PE guidelines

All contents of this site are ©2008 passmedicine.com - Terms and Conditions
passmedicine.compassmedicine.comReference ranges End session

Question 171 of 1500


Which one of the following types of lung cancer is most associated with
cavitating lesions?ia A.ACarcinoidia
B.ALarge cellia
C.ASmall cellia
D.ASquamous cellia
E.AAdenocarcinomaia

Whilst the other types of lung cancer may cause cavitating lesions, it is most
commonly seen in squamous cell cancer

CXR: cavitating lung lesion


sqweqwesf erwrewfsdfs adasd dhe
Differential
tuberculosis
lung cancer (especially squamous cell)
abscess (Staph aureus, Klebsiella and Pseudomonas)
Wegener's granulomatosis
pulmonary embolism
rheumatoid arthritis
aspergillosis, histoplasmosis, coccidioidomycosis

Question stats

A9%
B7.2%
C12.8%
D55.4%
E15.6%

55.4% of users answered this question correctly


All contents of this site are ©2008 passmedicine.com - Terms and Conditions
passmedicine.compassmedicine.comReference ranges End session

Question 194 of 1500


A 62-year-old female with a 40 pack year history of smoking is investigated for
a chronic cough associated with haemoptysis. Bronchoscopy reveals a small 1 cm
tumour confined to the right main bronchus. A biopsy taken shows small cell lung
cancer. What is the most appropriate management?ia A.ALaser therapyia
B.AChemotherapyia
C.ASurgeryia
D.ARadiotherapyia
E.AInterferon-alphaia

Surgery plays little role in the management of small cell lung cancer, with
chemotherapy being the mainstay of treatment

Lung cancer: small cell


sqweqwesf erwrewfsdfs adasd dhe
Features

usually central
arise from APUD cells
associated with ectopic ADH, ACTH secretion
ADH --> hyponatraemia
ACTH --> Cushing's syndrome
ACTH secretion can cause bilateral adrenal hyperplasia, the high levels of
cortisol can lead to hypokalaemic alkalosis
Lambert-Eaton syndrome: antibodies to voltage gated calcium channels causing
myasthenic like syndrome

SCcHhG: ‫ سجق‬siadh, cushing, carcinoid, hypercalcemia, hypocalcemia, gynecomastia


Management

usually metastatic disease by time of diagnosis


surgery: only used for debulking
radiotherapy: only used for debulking
chemotherapy: good response to combination chemotherapy, may extend life by
approximately 4 months

Question stats

A2.2%
B61%
C19.6%
D16.8%
E0.5%

61% of users answered this question correctly

May 2008 exam


External links

SIGN
Lung cancer management guidelines

All contents of this site are ©2008 passmedicine.com - Terms and Conditions
passmedicine.compassmedicine.comReference ranges End session

Question 202 of 1500


A chest x-ray of a patient with sarcoidosis shows bilateral hilar
lymphadenopathy but is otherwise normal. What chest x-ray stage does this
correspond to?ia A.AStage 0ia
B.AStage 1ia
C.AStage 2ia
D.AStage 3ia
E.AStage 4ia

Sarcoidosis CXR

1 = BHL
2 = BHL + infiltrates
3 = infiltrates
4 = fibrosis

Sarcoidosis: investigation
sqweqwesf erwrewfsdfs adasd dhe
There is no one diagnostic test for sarcoidosis and hence diagnosis is still
largely clinical. ACE levels have a sensitivity of 60% and specificity of 70%
and are therefore not reliable in the diagnosis of sarcoidosis although they may
have a role in monitoring disease activity. Routine bloods may show
hypercalcaemia (seen in 10% if patients) and a raised ESR

A chest x-ray may show the following changes:

stage 0 = normal
stage 1 = bilateral hilar lymphadenopathy (BHL)
stage 2 = BHL + interstitial infiltrates
stage 3 = diffuse interstitial infiltrates only
stage 4 = diffuse fibrosis

Other investigations*

spirometry: may show a restrictive defect


tissue biopsy: non-caseating granulomas
gallium-67 scan - not used routinely

*the Kveim test (where part of the spleen from a patient with known sarcoidosis
is injected under the skin) is no longer performed due to concerns about
cross-infection

Question stats

A3.1%
B69%
C22.1%
D5%
E0.7%

69% of users answered this question correctly


All contents of this site are ©2008 passmedicine.com - Terms and Conditions
passmedicine.compassmedicine.comReference ranges End session

Question 208 of 1500


A 65-year-old female with a history of chronic obstructive pulmonary disease
(COPD) is reviewed in the Emergency Department. She has presented with a sudden
worsening of her dyspnoea associated with haemoptysis. What is the most suitable
initial imaging investigation to exclude a pulmonary embolism?ia
A.AVentilation-perfusion scania
B.AEchocardiogramia
C.APulmonary angiographyia
D.AComputed tomographic pulmonary angiographyia
E.AMRI thoraxia

It is still common in UK hospitals, despite guidelines, for a


ventilation-perfusion scan to be done first-line

Pulmonary embolism: investigation


sqweqwesf erwrewfsdfs adasd dhe
The British Thoracic Society (BTS) published guidelines in 2003 on the
management of patients with suspected pulmonary embolism (PE)

Key points from the guidelines include:

computed tomographic pulmonary angiography (CTPA) is now the recommended


initial lung-imaging modality for non-massive PE. Advantages compared to V/Q
scans include speed, easier to perform out-of-hours, a reduced need for
further imaging and the possibility of providing an alternative diagnosis if
PE is excluded
if the CTPA is negative then patients do not need further investigations or
treatment for PE
ventilation-perfusion scanning may be used initially if appropriate facilities
exist, the chest x-ray is normal, and there is no significant symptomatic
concurrent cardiopulmonary disease
Some other points

Clinical probability scores based on risk factors and history and now widely
used to help decide on further investigation/management

D-dimers

sensitivity = 95-98%, but poor specificity

V/Q scan

sensitivity = 98%; specificity = 40% - high negative predictive value, i.e. if


normal virtually excludes PE
other causes of mismatch in V/Q include old pulmonary embolisms, AV
malformations, vasculitis, previous radiotherapy
COPD gives matched defects

CTPA

peripheral emboli affecting subsegmental arteries may be missed

Pulmonary angiography

the gold standard


significant complication rate compared to other investigations

Question stats

A15%
B2.4%
C4.5%
D76.8%
E1.3%

76.8% of users answered this question correctly

May 2007 exam


External links

Postgraduate Medical Journal


Diagnosing pulmonary embolism

British Thoracic Society


2003 PE guidelines

All contents of this site are ©2008 passmedicine.com - Terms and Conditions
passmedicine.compassmedicine.comReference ranges End session

Question 215 of 1500


A 45-year-old man is noted to have bilateral hilar lymphadenopathy on chest
x-ray. Which one of the following is the least likely cause?ia
A.AAmyloidosisia
B.ASarcoidosisia
C.AHistoplasmosisia
D.ATuberculosisia
E.ABerylliosisia
Amyloidosis is not commonly associated with bilateral hilar lymphadenopathy

Bilateral hilar lymphadenopathy


sqweqwesf erwrewfsdfs adasd dhe
The most common causes of bilateral hilar lymphadenopathy are sarcoidosis and
tuberculosis

Other causes include:

lymphoma/other malignancy
pneumoconiosis e.g. berylliosis
fungi e.g. histoplasmosis, coccidioidomycosis

Question stats

A54.4%
B6.1%
C13.3%
D8.4%
E17.8%

54.4% of users answered this question correctly


All contents of this site are ©2008 passmedicine.com - Terms and Conditions
passmedicine.compassmedicine.comReference ranges End session

Question 227 of 1500


A 24-year-old male with no past medical history presents to the Emergency
Department with pleuritic chest pain. There is no history of a productive cough
and he is not short of breath. Chest x-ray shows a right-sided pneumothorax with
a 1.5cm rim of air and no mediastinal shift. What is the most appropriate
management?ia A.AImmediate 14G cannula into 2nd intercostal space,
mid-clavicular lineia
B.ADischargeia
C.AAspirationia
D.AIntercostal drain insertionia
E.AAdmit for 48 hours observationia

It was of course be prudent to give advice about what he should do if his


symptoms worsen and also suggest routine follow-up with his GP

Pneumothorax
sqweqwesf erwrewfsdfs adasd dhe
The British Thoracic Society (BTS) published guidelines for the management of
spontaneous pneumothorax in 2003. A pneumothorax is termed primary if there is
no underlying lung disease and secondary if there is

Primary pneumothorax

Recommendations include:

if the rim of air is < 2cm and the patient is not short of breath then
discharge should be considered
otherwise aspiration should be attempted
if this fails then repeat aspiration should be considered
if this fails then a chest drain should be inserted
Secondary pneumothorax

Recommendations include:

if the patient is > 50 years old and the rim of air is > 2cm and the patient
is short of breath then a chest drain should be inserted.
otherwise aspiration should be attempted. If aspiration fails a chest drain
should be inserted. All patients should be admitted for at least 24 hours

Iatrogenic pneumothorax

Recommendations include:

less likelihood of recurrence than spontaneous pneumothorax


majority will resolve with observation, if treatment is required then
aspiration should be used
ventilated patients need chest drains, as may some patients with COPD

Question stats

A3.9%
B53.4%
C13.5%
D2.7%
E26.4%

53.4% of users answered this question correctly


External links

British Thoracic Society


Pneumothorax guidelines

All contents of this site are ©2008 passmedicine.com - Terms and Conditions
passmedicine.compassmedicine.comReference ranges End session

Question 234 of 1500


Which one of the following paraneoplastic features is least commonly seen in
patients with squamous cell lung cancer?ia A.ALambert-Eaton syndromeia
B.AHyperthyroidismia
C.AHypertrophic pulmonary osteoarthropathyia
D.AHypercalcaemiaia
E.AClubbingia

Paraneoplastic features of lung cancer

squamous cell: PTHrp, clubbing, HPOA


small cell: ADH, ACTH, Lambert-Eaton syndrome

Lambert-Eaton syndrome occurs almost exclusively in small cell lung cancer

Lung cancer: paraneoplastic features


sqweqwesf erwrewfsdfs adasd dhe
Small cell
ADH
ACTH - not typical, hypertension, hyperglycaemia, hypokalaemia, alkalosis and
muscle weakness are more common than buffalo hump etc
Lambert-Eaton syndrome

Squamous cell

PTH-rp
clubbing
hypertrophic pulmonary osteoarthropathy (HPOA)
hyperthyroidism due to ectopic TSH

Adenocarcinoma

gynaecomastia

Question stats

A47.5%
B34.3%
C7.8%
D5.1%
E5.3%

47.5% of users answered this question correctly


All contents of this site are ©2008 passmedicine.com - Terms and Conditions
passmedicine.compassmedicine.comReference ranges End session

Question 308 of 1500


A 55-year-old man is referred to the medical admissions unit. He recently
returned from a holiday in Italy and has failed to respond to a course of
co-amoxiclav for a suspected lower respiratory tract infection. Chest x-ray
shows bilateral infiltrates. Bloods are as follows:

Na+122 mmol/l
K+4.3 mmol/l
Urea8.4 mmol/l
Creatinine130 µmol/l

What is the likely diagnosis?ia A.AGoodpasture's syndromeia


B.ALegionella pneumoniaia
C.APneumocystis carinii pneumoniaia
D.APulmonary eosinophiliaia
E.AMycoplasma pneumoniaia

Legionella
sqweqwesf erwrewfsdfs adasd dhe
Legionnaire's disease is caused by the intracellular bacterium Legionella
pneumophilia. It is typically colonizes water tanks and hence questions may hint
at air-conditioning systems or foreign holidays. Person-to-person transmission
is not seen

Features
flu-like symptoms
dry cough
lymphopenia
hyponatraemia
deranged LFTs

Diagnosis

urinary antigen

Management

treat with erythromycin

Question stats

A5.2%
B77.8%
C2.6%
D0.7%
E13.7%

77.8% of users answered this question correctly

May 2006 exam


All contents of this site are ©2008 passmedicine.com - Terms and Conditions
passmedicine.compassmedicine.comReference ranges End session

Question 320 of 1500


A 52-year-old woman with a history of breast cancer is admitted with acute
dyspnoea. Her respiratory rate on admission is 42 / min and her oxygen
saturations are 87% on room air. A pulmonary embolism is suspected and she is
transferred to the high dependency unit after being treated with oxygen and
enoxaparin. Which one of the following would be strongest indication for
thrombolysis?ia A.AExtensive deep venous thrombosisia
B.AHypotensionia
C.APatient choice following informed consentia
D.AHypoxaemia despite high flow oxygenia
E.AECG showing right ventricular strainia

Massive PE + hypotension - thrombolyse

Pulmonary embolism: management


sqweqwesf erwrewfsdfs adasd dhe
The British Thoracic Society (BTS) published guidelines in 2003 on the
management of patients with suspected pulmonary embolism (PE)

Key points from the guidelines include:

Anticoagulation
low molecular weight heparin (LMWH), rather than unfractionated heparin (UFH),
should be used routinely in patients with suspected pulmonary embolism. This
reflects the equal efficacy and safety of LMWHs as well as their ease of use.
Exceptions include patients with a massive PE or in situations where rapid
reversal of anticoagulation may be necessary
warfarinisation: the standard duration of anticoagulation is 4-6 weeks* if
temporary risk factors are present, 3 months for the first idiopathic PE and
at least 6 months for other situations

Thrombolysis

thrombolysis is now recommended as the first-line treatment for massive PE


where there is circulatory failure (e.g. hypotension). Other invasive
approaches should be considered where appropriate facilities exist

*the 2005 British Committee for Standards in Haematology guidelines suggest at


least 3 months anticoagulation in this situation, based on a 2004 study. It is
therefore possible these guidelines may soon change

Question stats

A7%
B57.1%
C1.7%
D20.5%
E13.8%

57.1% of users answered this question correctly

September 2008 exam


External links

British Thoracic Society


2003 PE guidelines

All contents of this site are ©2008 passmedicine.com - Terms and Conditions
passmedicine.compassmedicine.comReference ranges End session

Question 350 of 1500


A 62-year-old female is admitted with a suspected infective exacerbation of
COPD. A chest x-ray shows no evidence of consolidation. What is the most likely
causative organism?ia A.APseudomonas aeruginosaia
B.AHaemophilus influenzaeia
C.AStaphylococcus aureusia
D.AStreptococcus pneumoniaeia
E.AMoraxella catarrhalisia

COPD: acute management


sqweqwesf erwrewfsdfs adasd dhe
The most bacterial common organisms that cause infective exacerbations of COPD
are:

Haemophilus influenzae (most common cause)


Streptococcus pneumoniae
Moraxella catarrhalis
Respiratory viruses account for around 30% of exacerbations, with the human
rhinovirus being the most important pathogen

Non-invasive ventilation has been shown to effective in the management of


patients with acute respiratory failure

Question stats

A10.2%
B48.6%
C3.8%
D17.4%
E19.9%

48.6% of users answered this question correctly

May 2006 exam


External links

Postgraduate Medical Journal


Acute COPD management

All contents of this site are ©2008 passmedicine.com - Terms and Conditions
passmedicine.compassmedicine.comReference ranges End session

Question 377 of 1500


A 24-year-old female with a history of anxiety is taken to the Emergency
Department following an acute onset of shortness of breath. On examination the
chest is clear to auscultation but the respiratory rate is raised at 40 breaths
per minute. A diagnosis of hyperventilation secondary to anxiety is suspected.
Which of the following arterial blood gas results (taken on room air) are
consistent with this?ia A.ApH = 7.56; pCO2 = 2.9 kPa; pO2 = 10.1 kPaia
B.ApH = 7.24; pCO2 = 8.4 kPa; pO2 = 12.7 kPaia
C.ApH = 7.34; pCO2 = 2.7 kPa; pO2 = 15.4 kPaia
D.ApH = 7.54; pCO2 = 2.4 kPa; pO2 = 14.1 kPaia
E.ApH = 7.54; pCO2 = 4.9 kPa; pO2 = 13.3 kPaia

Hyperventilation will result in carbon dioxide being 'blown off', causing an


alkalosis.

Whilst the gases in answer A show a respiratory alkalosis the hypoxia could not
be explained by hyperventilation

Respiratory alkalosis
sqweqwesf erwrewfsdfs adasd dhe
Common causes

anxiety leading to hyperventilation


pulmonary embolism
salicylate poisoning*
CNS disorders: stroke, subarachnoid haemorrhage, encephalitis
altitude
pregnancy

*salicylate overdose leads to a mixed respiratory alkalosis and metabolic


acidosis. Early stimulation of the respiratory centre leads to a respiratory
alkalosis whilst later the direct acid effects of salicylates (combined with
acute renal failure) may lead to an acidosis

Question stats

A13.7%
B2.9%
C9.8%
D68.9%
E4.7%

68.9% of users answered this question correctly

May 2006 exam


All contents of this site are ©2008 passmedicine.com - Terms and Conditions
passmedicine.compassmedicine.comReference ranges End session

Question 388 of 1500


Which one of the following is least associated with Kartagener's syndrome?ia
A.AMale subfertilityia
B.ARecurrent sinusitisia
C.AMalabsorptionia
D.ADextrocardiaia
E.ABronchiectasisia

Kartagener's syndrome
sqweqwesf erwrewfsdfs adasd dhe
Kartagener's syndrome (also known as primary ciliary dyskinesia) was first
described in 1933 and most frequently occurs in examinations due to its
association with dextrocardia (e.g. 'quiet heart sounds', 'small volume
complexes in lateral leads')

Features

dextrocardia or complete situs inversus


bronchiectasis
recurrent sinusitis
subfertility (secondary to diminished sperm motility and defective ciliary
action in the fallopian tubes)

Question stats

A11.8%
B8.8%
C59.5%
D16.4%
E3.5%

59.5% of users answered this question correctly


All contents of this site are ©2008 passmedicine.com - Terms and Conditions
passmedicine.compassmedicine.comReference ranges End session
Question 400 of 1500
Each one of the following may result in bronchiectasis, except:ia
A.AKartagener's syndromeia
B.AAmyloidosisia
C.ASelective IgA deficiencyia
D.ALung canceria
E.AAllergic bronchopulmonary aspergillosisia

Amyloidosis does not cause bronchiectasis per se, but may be seen in
bronchiectasis as a consequence of chronic inflammation and infection

Bronchiectasis: causes
sqweqwesf erwrewfsdfs adasd dhe
Bronchiectasis describes a permanent dilatation of the airways secondary to
chronic infection or inflammation. There are a wide variety of causes are listed
below:

Causes

post-infective: tuberculosis, measles, pertussis, pneumonia


cystic fibrosis
bronchial obstruction e.g. lung cancer/foreign body
immune deficiency: selective IgA, hypogammaglobulinaemia
allergic bronchopulmonary aspergillosis (ABPA)
ciliary dyskinetic syndromes: Kartagener's syndrome, Young's syndrome
yellow nail syndrome

Question stats

A3.5%
B63.5%
C11.2%
D13.7%
E8.2%

63.5% of users answered this question correctly


All contents of this site are ©2008 passmedicine.com - Terms and Conditions
passmedicine.compassmedicine.comReference ranges End session

Question 434 of 1500


A 31-year-old woman is referred to the acute medical unit with a 4 day history
of polyarthritis and a low-grade pyrexia. Examination reveals shin lesions which
the patient states are painful. Chest x-ray shows a bulky mediastinum. What is
the most likely diagnosis?ia A.ALoffler's syndromeia
B.ALofgren's syndromeia
C.ASystemic lupus erythematousia
D.AGonococcal arthritisia
E.AReiter's syndromeia

Loffler's syndrome is a cause of pulmonary eosinophilia thought to be caused by


parasites such as Ascaris lumbricoides

Lofgren's syndrome
sqweqwesf erwrewfsdfs adasd dhe
Lofgren's syndrome is an acute form sarcoidosis characterised by bilateral hilar
lymphadenopathy (BHL), erythema nodosum, fever and polyarthralgia.

It typically occurs in young females and carries an excellent prognosis

Question stats

A22.4%
B52.5%
C17.3%
D2.7%
E5.1%

52.5% of users answered this question correctly


All contents of this site are ©2008 passmedicine.com - Terms and Conditions
passmedicine.compassmedicine.comReference ranges End session

Question 489 of 1500


A 52-year-old man is admitted to hospital with breathlessness and fever. He has
no significant past medical history of note and has not seen a doctor for over 5
years. On examination the temperature is 38.4ºC, respiratory rate is 24 / min,
pulse is 84 / min and the blood pressure is 118/72 mmHg. A chest x-ray reveals
right lower lobe consolidation. Arterial blood gases on air are as follows:

pH7.39
pCO24.6 kPa
pO29.8 kPa

What is the most suitable antibiotic therapy?ia A.AOral co-amoxiclavia


B.AOral amoxicillin + erythromycinia
C.AOral amoxicillinia
D.AIV ceftriaxone + clarithromycinia
E.AOral co-amoxiclav + metronidazoleia

Uncomplicated - give only amoxicillin if:

treated in the community, or


admitted to hospital for non-clinical reasons, or
not previously treated in the community

The 2004 British Thoracic Society guidelines recommend oral amoxicillin as the
first-line antibiotic for hospitalised patients with non-severe community
acquired pneumonia, if they have been admitted for non-clinical reasons or have
not previously been treated in the community.

Pneumonia: community-acquired
sqweqwesf erwrewfsdfs adasd dhe
Community acquired pneumonia (CAP) may be caused by the following organisms:

Streptococcus pneumoniae (accounts for around 80% of cases)


Haemophilus influenzae
Staphylococcal aureus
atypical pneumonias (e.g. due to Mycoplasma pneumoniae)
viruses

Streptococcus pneumoniae (pneumococcus) is the most common cause of


community-acquired pneumonia
Characteristic features of pneumococcal pneumonia

rapid onset
high fever
pleuritic chest pain
herpes labialis

Antibiotic choices

home-treated uncomplicated CAP: first line - oral amoxicillin


hospitalized uncomplicated CAP: if admitted for non-clinical reasons or not
previously treated in the community then oral amoxicillin, otherwise
amoxicillin + macrolide

Question stats

A12.5%
B28.3%
C46.1%
D11.6%
E1.4%

46.1% of users answered this question correctly

May 2004 exam


External links

British Thoracic Society


2004 CAP guidelines update

All contents of this site are ©2008 passmedicine.com - Terms and Conditions
passmedicine.compassmedicine.comReference ranges End session

Question 491 of 1500


A 24-year-old male is admitted with acute severe asthma. Treatment is initiated
with 100% oxygen, nebulised salbutamol and ipratropium bromide nebulisers and IV
hydrocortisone. Despite initial treatment there is no improvement. What is the
next step in management?ia A.AIV aminophyllineia
B.AIV magnesium sulphateia
C.AIV salbutamolia
D.AIM adrenalineia
E.AIV adrenalineia

Asthma: acute severe


sqweqwesf erwrewfsdfs adasd dhe
Patients with acute severe asthma are stratified into moderate, severe or
life-threatening

ModerateSevereLife-threatening
• PEF > 50% best or predicted
• Speech normal
• RR < 25 / min
• Pulse < 110 bpm• PEF 33 - 50% best or predicted
• Can't complete sentences
• RR > 25/min
• Pulse > 110 bpm• PEF < 33% best or predicted
• Oxygen sats < 92%
• Silent chest, cyanosis or feeble respiratory effort
• Bradycardia, dysrhythmia or hypotension
• Exhaustion, confusion or coma

British Thoracic Society guidelines 2003 (updated 2004)

magnesium sulphate recommended as next step for patients who are not
responding (e.g. 1.2 - 2g IV over 20 mins)
little evidence to support use of IV aminophylline (although still mentioned
in management plans)
if no response consider IV salbutamol

Question stats

A17.8%
B74.2%
C5%
D1.8%
E1.2%

74.2% of users answered this question correctly

September 2008 exam


External links

British Thoracic Society


2008 Asthma guidelines

All contents of this site are ©2008 passmedicine.com - Terms and Conditions
passmedicine.compassmedicine.comReference ranges End session

Question 494 of 1500


Which one of the following interventions is most likely to increase survival in
patients with COPD?ia A.AHome nebulisersia
B.AProphylactic antibiotic therapyia
C.APulmonary rehabilitationia
D.ALong-term steroid therapyia
E.ALong-term oxygen therapyia

After smoking cessation, long-term oxygen therapy is one of the few


interventions that has been shown to improve survival in COPD

COPD: stable management


sqweqwesf erwrewfsdfs adasd dhe
General management

1 smoking cessation advice


pneumococcal vaccine
annual influenza vaccine

2Bronchodilator therapy
short acting beta2-agoinst or anticholinergic is first-line treatment
if still symptomatic add a long-acting anti-cholinergic (e.g. tiotropium) or a
long-acting beta2-agonist (e.g. salmeterol)

3 Inhaled steroids

NICE recommends that an inhaled corticosteroid should for patients with an


FEV1 < 50% of predicted (i.e. moderate or severe COPD), who are having two or
more exacerbations requiring treatment with antibiotics or oral
corticosteroids in a 12 month period
reduce frequency of exacerbations

If the patient is symptomatic despite the above measures

4oral theophylline

Factors which may improve survival in patients with stable COPD

smoking cessation - the single most important intervention in patients who are
still smoking
long term oxygen therapy in patients who fit criteria
lung volume reduction surgery in selected patients

Question stats

A2%
B2%
C15.7%
D2.3%
E78%

78% of users answered this question correctly

January 2006 exam


External links

NICE
COPD guidelines

Postgraduate Medical Journal


Acute COPD management

All contents of this site are ©2008 passmedicine.com - Terms and Conditions
passmedicine.compassmedicine.comReference ranges End session

Question 498 of 1500


A 67-year-old female is referred to the acute medical unit with an infective
exacerbation of COPD. Despite maximal medical therapy the arterial blood gases
continue to show type II respiratory failure. You are asked to consider
non-invasive ventilation. At what pH is the patient most likely to receive
benefit from non-invasive ventilation?ia A.ApH 7.13ia
B.ApH 7.18ia
C.ApH 7.23ia
D.ApH 7.29ia
E.ApH 7.37ia
The evidence surrounding the use of NIV in COPD shows that patients with a pH in
the range of 7.25-7.35 achieve the most benefit. If the pH is < 7.25 then
invasive ventilation should be considered if appropriate

Non-invasive ventilation
sqweqwesf erwrewfsdfs adasd dhe
The British Thoracic Society (BTS) published guidelines in 2002 on the use of
non-invasive ventilation in acute respiratory failure

Non-invasive ventilation - key indications

COPD with respiratory acidosis pH 7.25-7.35


type II respiratory failure secondary to chest wall deformity, neuromuscular
disease or obstructive sleep apnoea
cardiogenic pulmonary oedema unresponsive to CPAP
weaning from tracheal intubation

Recommended initial settings for bi-level pressure support in COPD

Expiratory Positive Airway Pressure (EPAP): 4-5 cm H2O


Inspiratory Positive Airway Pressure (IPAP): RCP advocate 10 cm H20 whilst BTS
suggest 12-15 cm H2O
back up rate: 15 breaths/min
back up inspiration:expiration ratio: 1:3

Question stats

A6.9%
B7.4%
C30.3%
D48.4%
E7.2%

48.4% of users answered this question correctly


External links

RCP
2008 NIV in COPD guidelines

British Thoracic Society


2002 NIV guidelines

All contents of this site are ©2008 passmedicine.com - Terms and Conditions
passmedicine.compassmedicine.comReference ranges End session

Question 506 of 1500


A 35-year-old female with sarcoidosis is started on a course of prednisolone.
Which one of the following is a suitable indication for commencing steroid
therapy in such patients?ia A.ABilateral hilar lymphadenopathyia
B.AArthralgiaia
C.AHypercalcaemiaia
D.ASerum ACE > 120 u/lia
E.AErythema nodosumia
Sarcoidosis: management
sqweqwesf erwrewfsdfs adasd dhe
Sarcoidosis is a multisystem disorder of unknown aetiology characterised by
non-caseating granulomas. It is more common in young adults and in people of
African descent

Indications for steroids

hypercalcaemia
worsening lung function
eye, heart or neuro involvement

not for skin, arthralgia, ACE rise

Question stats

A10.2%
B12.9%
C55.9%
D9%
E12%

55.9% of users answered this question correctly

January 2006 exam


All contents of this site are ©2008 passmedicine.com - Terms and Conditions
passmedicine.compassmedicine.comReference ranges End session

Question 544 of 1500


Which one the following statements regarding asbestos is not correct?ia
A.APleural plaques are premalignantia
B.AAsbestosis typically affects the lower zonesia
C.ACrocidolite (blue) asbestos is the most dangerous formia
D.ASeverity of asbestosis is related to the length of exposureia
E.AMesothelioma may develop following minimal exposureia

Asbestos
sqweqwesf erwrewfsdfs adasd dhe
The severity of asbestosis is related to the length of exposure. This is in
contrast to mesothelioma where even very limited exposure can cause disease.
Asbestosis typically causes lower lobe fibrosis. Crocidolite (blue) asbestos is
the most dangerous form

Other features

pleural thickening
pleural plaques also seen (not premalignant)

Question stats
A48%
B14%
C10.8%
D12.1%
E15.2%

48% of users answered this question correctly


All contents of this site are ©2008 passmedicine.com - Terms and Conditions
passmedicine.compassmedicine.comReference ranges End session

Question 554 of 1500


Which one of the following is least associated with the development of COPD?ia
A.ACadmium exposureia
B.ASmokingia
C.ACoal dustia
D.AIsocyanatesia
E.AAlpha-1 antitrypsin deficiencyia

Isocyanates are more associated with occupational asthma

COPD: causes
sqweqwesf erwrewfsdfs adasd dhe
Smoking!

Alpha-1 antitrypsin deficiency

Other causes

cadmium (used in smelting)


coal
cotton
cement
grain

Question stats

A29.9%
B0.5%
C19.7%
D41.3%
E8.5%

41.3% of users answered this question correctly


All contents of this site are ©2008 passmedicine.com - Terms and Conditions
passmedicine.compassmedicine.comReference ranges End session

Question 579 of 1500


A 39-year-old man presents with shortness of breath following one week of
flu-like symptoms. He also has a non-productive but no chest pain. A chest x-ray
shows bilateral consolidation and examination reveals erythematous lesions on
his limbs and trunk. Which one of the following investigations is most likely to
be diagnostic?ia A.ACold agglutinsia
B.ASputum cultureia
C.AUrinary antigen for Legionellaia
D.ASerology for Mycoplasmaia
E.ABlood cultureia

Mycoplasma? - serology is diagnostic

The flu-like symptoms, bilateral consolidation and erythema multiforme point to


a diagnosis of Mycoplasma. The most appropriate diagnostic test is Mycoplasma
serology

Mycoplasma pneumoniae
sqweqwesf erwrewfsdfs adasd dhe
Mycoplasma pneumoniae is a cause of atypical pneumonia which often affects
younger patients. It is associated with a number of characteristic complications
such as erythema multiforme and cold autoimmune haemolytic anaemia. Epidemics of
Mycoplasma pneumoniae classically occur every 4 years. It is important to
recognise atypical pneumonias as they may not respond to penicillins or
cephalosporins

Features

flu-like symptoms classically precede a dry cough


bilateral consolidation on x-ray
complications may occur as below

Complications

cold agglutins (IgM) may cause an haemolytic anaemia, thrombocytopenia


erythema multiforme, erythema nodosum
meningoencephalitis, Guillain-Barre syndrome
bullous myringitis: painful vesicles on the tympanic membrane
pericarditis/myocarditis
gastrointestinal: hepatitis, pancreatitis
renal: acute glomerulonephritis

Diagnosis

Mycoplasma serology

Management

erythromycin/clarithromycin

Question stats

A16.2%
B1.7%
C13.8%
D66.2%
E2.1%

66.2% of users answered this question correctly

September 2008 exam


All contents of this site are ©2008 passmedicine.com - Terms and Conditions
passmedicine.compassmedicine.comReference ranges End session

Question 586 of 1500


Which type of hypersensitivity reaction predominates in the acute phase of
extrinsic allergic alveolitis?ia A.AType Iia
B.AType IIia
C.AType IIIia
D.AType IVia
E.AType Via

Although it is known that the pathogenesis of extrinsic allergic alveolitis


involves a type IV (delayed) hypersensitivity reaction, a type III
hypersensitivity reaction is thought to predominate, especially in the acute
phase

Extrinsic allergic alveolitis


sqweqwesf erwrewfsdfs adasd dhe
Extrinsic allergic alveolitis (EAA) is a condition caused by hypersensitivity
induced lung damage due to a variety of inhaled organic particles. It is thought
to be largely caused by immune-complex mediated tissue damage (type III
hypersensitivity) although delayed hypersensitivity (type IV) is also thought to
play a role in EAA, especially in the chronic phase

Examples

bird fanciers' lung (avian proteins)


farmers lung (spores of Micropolyspora faeni)
malt workers' lung (Aspergillus clavatus)
mushroom workers' lung (thermophilic actinomycetes*)

Presentation

acute: occur 4-8 hrs after exposure, SOB, dry cough, fever
chronic

Investigation

CXR: upper lobe fibrosis


BAL: lymphocytosis
blood: NO eosinophilia

*here the terminology is slightly confusing as thermophilic actinomycetes is an


umbrella term covering strains such as Micropolyspora faeni

Question stats

A11.6%
B18%
C54%
D15.2%
E1.2%

54% of users answered this question correctly

May 2005 exam


All contents of this site are ©2008 passmedicine.com - Terms and Conditions
passmedicine.compassmedicine.comReference ranges End session

Question 631 of 1500


A 60-year-old woman who has recently been diagnosed with chronic obstructive
pulmonary disease (COPD) presents for review. She is still occasionally
breathless despite bronchodilator therapy. Which one of the following criteria
is most relevant when deciding who would benefit from inhaled corticosteroids?ia
A.AFailure of long-acting beta2-agonist to improve breathlessnessia
B.AFEV1/FVC < 50% predictedia
C.AFEV1 < 50% predicted and 2 or more exacerbations per yearia
D.AFEV1/FVC < 50% predicted and 3 or more exacerbations per yearia
E.AFour or more than exacerbations per yearia

COPD: stable management


sqweqwesf erwrewfsdfs adasd dhe
General management

smoking cessation advice


pneumococcal vaccine
annual influenza vaccine

Bronchodilator therapy

short acting beta2-agoinst or anticholinergic is first-line treatment


if still symptomatic add a long-acting anti-cholinergic (e.g. tiotropium) or a
long-acting beta2-agonist (e.g. salmeterol)

Inhaled steroids

NICE recommends that an inhaled corticosteroid should for patients with an


FEV1 < 50% of predicted (i.e. moderate or severe COPD), who are having two or
more exacerbations requiring treatment with antibiotics or oral
corticosteroids in a 12 month period
reduce frequency of exacerbations

If the patient is symptomatic despite the above measures

oral theophylline

Factors which may improve survival in patients with stable COPD

smoking cessation - the single most important intervention in patients who are
still smoking
long term oxygen therapy in patients who fit criteria
lung volume reduction surgery in selected patients

Question stats

A17.2%
B9.6%
C51.7%
D14%
E7.4%

51.7% of users answered this question correctly


External links

NICE
COPD guidelines

Postgraduate Medical Journal


Acute COPD management

All contents of this site are ©2008 passmedicine.com - Terms and Conditions
passmedicine.compassmedicine.comReference ranges End session

Question 644 of 1500


A 62-year-old man with a history of recurrent lower respiratory tract infections
is diagnosed as having bilateral bronchiectasis following a high resolution CT
scan. What is the treatment of choice for symptom control?ia A.APostural
drainageia
B.AProphylactic antibioticsia
C.ASurgeryia
D.AInspiratory muscle trainingia
E.AMucolytic therapyia

Symptom control in non-CF bronchiectasis - physical training rather than


postural drainage

This is a very tough question as all of the above options have been used in the
management of bronchiectasis. However, the only option with an evidence base is
physical training (e.g. inspiratory muscle training) for patients with
non-cystic fibrosis bronchiectasis.

Bronchiectasis: management
sqweqwesf erwrewfsdfs adasd dhe
Bronchiectasis describes a permanent dilatation of the airways secondary to
chronic infection or inflammation. After assessing for treatable causes (e.g.
immune deficiency) management is as follows:

physical training (e.g. inspiratory muscle training)


postural drainage
antibiotics for exacerbations + long-term rotating antibiotics in severe cases
bronchodilators in selected cases
immunisations
surgery in selected cases (e.g. localised disease)

Question stats

A46.7%
B10%
C1.7%
D31.6%
E10%

31.6% of users answered this question correctly

May 2006 exam


All contents of this site are ©2008 passmedicine.com - Terms and Conditions
passmedicine.compassmedicine.comReference ranges End session

Question 648 of 1500


A 58-year-old man is investigated for a chronic cough and is found to have lung
cancer. He enquires whether it may be work related. Which one of the following
is most likely to increase his risk of developing lung cancer?ia
A.AIsocyanatesia
B.ASoldering flux resinia
C.APassive smokingia
D.ACoal dustia
E.APolyvinyl chlorideia

Whilst many chemicals have been implicated in the development of lung cancer
passive smoking is the most likely cause. Up to 15% of lung cancers in patients
who do not smoke are thought to be caused by passive smoking

Lung cancer: risk factors


sqweqwesf erwrewfsdfs adasd dhe
Smoking

increases risk of lung ca by a factor of 10

Other factors

asbestos - increases risk of lung ca by a factor of 5


arsenic
radon
nickel
chromate
aromatic hydrocarbon
cryptogenic fibrosing alveolitis

Factors that are NOT related

coal dust

Smoking and asbestos are synergistic, i.e. a smoker with asbestos exposure has a
10 * 5 = 50 times increased risk

Question stats

A11.8%
B6.4%
C58.9%
D12.9%
E10.1%

58.9% of users answered this question correctly

May 2005 exam


All contents of this site are ©2008 passmedicine.com - Terms and Conditions
passmedicine.compassmedicine.comReference ranges End session

Question 653 of 1500


A 31-year-old female with no past medical history of note is admitted to
hospital with dyspnoea and fever. She has recently returned from holiday in
Turkey. A clinical diagnosis of pneumonia is made. On examination she is noted
to have an ulcerated lesion on her upper lip consistent with reactivation of
herpes simplex. Which organism is most associated with this examination
finding?ia A.ALegionella pneumophiliaia
B.AStaphylococcal aureusia
C.AStreptococcus pneumoniaeia
D.APneumocystis cariniiia
E.AMycoplasma pneumoniaeia

Streptococcus pneumoniae commonly causes reactivation of the herpes simplex


virus resulting in 'cold sores'

Pneumonia: community-acquired
sqweqwesf erwrewfsdfs adasd dhe
Community acquired pneumonia (CAP) may be caused by the following organisms:

Streptococcus pneumoniae (accounts for around 80% of cases)


Haemophilus influenzae
Staphylococcal aureus
atypical pneumonias (e.g. due to Mycoplasma pneumoniae)
viruses

Streptococcus pneumoniae (pneumococcus) is the most common cause of


community-acquired pneumonia

Characteristic features of pneumococcal pneumonia

rapid onset
high fever
pleuritic chest pain
herpes labialis

Antibiotic choices

home-treated uncomplicated CAP: first line - oral amoxicillin


hospitalized uncomplicated CAP: if admitted for non-clinical reasons or not
previously treated in the community then oral amoxicillin, otherwise
amoxicillin + macrolide

Question stats

A24%
B7%
C46.4%
D10.3%
E12.3%

46.4% of users answered this question correctly

May 2008 exam


External links

British Thoracic Society


2004 CAP guidelines update

All contents of this site are ©2008 passmedicine.com - Terms and Conditions
passmedicine.compassmedicine.comReference ranges End session

Question 663 of 1500


Which one of the following causes of lung fibrosis predominately affect the
lower zones?ia A.AMethotrexateia
B.ASarcoidosisia
C.ACoal worker's pneumoconiosisia
D.AAnkylosing spondylitisia
E.AExtrinsic allergic alveolitisia

Lung fibrosis
sqweqwesf erwrewfsdfs adasd dhe
It is important in the exam to be able to differentiate between conditions
causing predominately upper or lower zone fibrosis. It should be noted that the
more common causes (cryptogenic fibrosing alveolitis, drugs) tend to affect the
lower zones

Fibrosis predominately affecting the upper zones

extrinsic allergic alveolitis


coal worker's pneumoconiosis/progressive massive fibrosis
silicosis
sarcoidosis
ankylosing spondylitis (rare)
histiocytosis
tuberculosis

Fibrosis predominately affecting the lower zones

cryptogenic fibrosing alveolitis


most connective tissue disorders (except ankylosing spondylitis)
drug-induced: amiodarone, bleomycin, methotrexate
asbestosis

Question stats

A66.6%
B4.7%
C10.3%
D7.5%
E10.8%

66.6% of users answered this question correctly


All contents of this site are ©2008 passmedicine.com - Terms and Conditions
passmedicine.compassmedicine.comReference ranges End session

Question 666 of 1500


A 30-year-old female with a past history of asthma presents to the Emergency
Department with shortness of breath. Chest x-ray shows a right-sided
pneumothorax with a 1.5cm rim of air and no mediastinal shift. What is the most
appropriate management?ia A.AAdmit for 48 hours observationia
B.AIntercostal drain insertionia
C.AAspirationia
D.ADischargeia
E.AImmediate 14G cannula into 2nd intercostal space, mid-clavicular
lineia

This should be treated as a secondary pneumothorax as the patient has a history


of asthma

Pneumothorax
sqweqwesf erwrewfsdfs adasd dhe
The British Thoracic Society (BTS) published guidelines for the management of
spontaneous pneumothorax in 2003. A pneumothorax is termed primary if there is
no underlying lung disease and secondary if there is

Primary pneumothorax

Recommendations include:

if the rim of air is < 2cm and the patient is not short of breath then
discharge should be considered
otherwise aspiration should be attempted
if this fails then repeat aspiration should be considered
if this fails then a chest drain should be inserted

Secondary pneumothorax

Recommendations include:

if the patient is > 50 years old and the rim of air is > 2cm and the patient
is short of breath then a chest drain should be inserted.
otherwise aspiration should be attempted. If aspiration fails a chest drain
should be inserted. All patients should be admitted for at least 24 hours

Iatrogenic pneumothorax

Recommendations include:

less likelihood of recurrence than spontaneous pneumothorax


majority will resolve with observation, if treatment is required then
aspiration should be used
ventilated patients need chest drains, as may some patients with COPD

Question stats

A17.3%
B5.9%
C49%
D24.8%
E3%

49% of users answered this question correctly


External links

British Thoracic Society


Pneumothorax guidelines

All contents of this site are ©2008 passmedicine.com - Terms and Conditions
passmedicine.compassmedicine.comReference ranges End session

Question 667 of 1500


Which of the following is not a recognised cause of pulmonary eosinophilia?ia
A.AChurg-Strauss syndromeia
B.ASulphonamidesia
C.AExtrinsic allergic alveolitisia
D.ALoffler's syndromeia
E.AAllergic bronchopulmonary aspergillosisia

Pulmonary eosinophilia
sqweqwesf erwrewfsdfs adasd dhe
Causes of pulmonary eosinophilia

Churg-Strauss syndrome
allergic bronchopulmonary aspergillosis (ABPA)
Loffler's syndrome
eosinophilic pneumonia
hypereosinophilic syndrome
tropical pulmonary eosinophilia
drugs: nitrofurantoin, sulphonamides
less common: Wegener's granulomatosis

Loffler's syndrome

transient CXR shadowing and blood eosinophilia


thought to be due to parasites such as Ascaris lumbricoides causing an
alveolar reaction
presents with a fever, cough and night sweats which often last for less than 2
weeks.
generally a self-limiting disease

Tropical pulmonary eosinophilia

associated with Wuchereria bancrofti infection

Question stats

A4.7%
B21.4%
C57.4%
D9.4%
E7.1%

57.4% of users answered this question correctly


All contents of this site are ©2008 passmedicine.com - Terms and Conditions
passmedicine.compassmedicine.comReference ranges End session

Question 703 of 1500


Which one of the following is least associated with bronchiectasis?ia
A.AHypogammaglobulinaemiaia
B.AAllergic bronchopulmonary aspergillosisia
C.AMeaslesia
D.ACystic fibrosisia
E.ASarcoidosisia

Traction bronchiectasis may be seen in some rare cases of stage IV pulmonary


sarcoidosis but this is the least strong association of the five options
Bronchiectasis: causes
sqweqwesf erwrewfsdfs adasd dhe
Bronchiectasis describes a permanent dilatation of the airways secondary to
chronic infection or inflammation. There are a wide variety of causes are listed
below:

Causes

post-infective: tuberculosis, measles, pertussis, pneumonia


cystic fibrosis
bronchial obstruction e.g. lung cancer/foreign body
immune deficiency: selective IgA, hypogammaglobulinaemia
allergic bronchopulmonary aspergillosis (ABPA)
ciliary dyskinetic syndromes: Kartagener's syndrome, Young's syndrome
yellow nail syndrome

Question stats

A7.9%
B8.5%
C24.1%
D1.7%
E57.7%

57.7% of users answered this question correctly


All contents of this site are ©2008 passmedicine.com - Terms and Conditions
passmedicine.compassmedicine.comReference ranges End session

Question 761 of 1500


A 54-year-old man is admitted with suspected pulmonary embolism. He has no past
medical history of note. Blood pressure is 120/80 mmHg with a pulse of 90/min.
The chest x-ray is normal. Following treatment with low-molecular weight
heparin, what is the most appropriate initial lung imaging investigation to
perform?ia A.APulmonary angiographyia
B.AEchocardiogramia
C.AMRI thoraxia
D.AVentilation-perfusion scania
E.AComputed tomographic pulmonary angiographyia

CTPA is the first line investigation for PE according to current BTS


guidelines

This is a difficult question to answer as both computed tomographic pulmonary


angiography (CTPA) and ventilation-perfusion scanning are commonly used in UK
clinical practice. The 2003 British Thoracic Society (BTS) guidelines, however,
recommended that CTPA is now used as the initial lung imaging modality of
choice. Pulmonary angiography is of course the 'gold standard' but this is not
what the question asks for

Pulmonary embolism: investigation


sqweqwesf erwrewfsdfs adasd dhe
The British Thoracic Society (BTS) published guidelines in 2003 on the
management of patients with suspected pulmonary embolism (PE)

Key points from the guidelines include:


computed tomographic pulmonary angiography (CTPA) is now the recommended
initial lung-imaging modality for non-massive PE. Advantages compared to V/Q
scans include speed, easier to perform out-of-hours, a reduced need for
further imaging and the possibility of providing an alternative diagnosis if
PE is excluded
if the CTPA is negative then patients do not need further investigations or
treatment for PE
ventilation-perfusion scanning may be used initially if appropriate facilities
exist, the chest x-ray is normal, and there is no significant symptomatic
concurrent cardiopulmonary disease

Some other points

Clinical probability scores based on risk factors and history and now widely
used to help decide on further investigation/management

D-dimers negative predictive value

sensitivity = 95-98%, but poor specificity

V/Q scan

sensitivity = 98%; specificity = 40% - high negative predictive value, i.e. if


normal virtually excludes PE
other causes of mismatch in V/Q include old pulmonary embolisms, AV
malformations, vasculitis, previous radiotherapy
COPD gives matched defects

CTPA

peripheral emboli affecting subsegmental arteries may be missed

Pulmonary angiography

the gold standard


significant complication rate compared to other investigations

Question stats

A2.3%
B0.8%
C0.9%
D21.3%
E74.8%

74.8% of users answered this question correctly


External links

Postgraduate Medical Journal


Diagnosing pulmonary embolism

British Thoracic Society


2003 PE guidelines

All contents of this site are ©2008 passmedicine.com - Terms and Conditions
passmedicine.compassmedicine.comReference ranges End session
Question 775 of 1500
A 43-year-old man is admitted due to shortness of breath and is noted to have a
cavitating lesion on his chest x-ray. Which one of the following conditions is
not part of the differential diagnosis?ia A.ALung canceria
B.APulmonary embolismia
C.AWegener's granulomatosisia
D.AChurg-Strauss syndromeia
E.ATuberculosisia

CXR: cavitating lung lesion


sqweqwesf erwrewfsdfs adasd dhe
Differential

tuberculosis
lung cancer (especially squamous cell)
abscess (Staph aureus, Klebsiella and Pseudomonas)
Wegener's granulomatosis
pulmonary embolism
rheumatoid arthritis
aspergillosis, histoplasmosis, coccidioidomycosis

Question stats

A1.2%
B34.4%
C5.1%
D58.7%
E0.6%

58.7% of users answered this question correctly


All contents of this site are ©2008 passmedicine.com - Terms and Conditions
passmedicine.compassmedicine.comReference ranges End session

Question 775 of 1500


A 43-year-old man is admitted due to shortness of breath and is noted to have a
cavitating lesion on his chest x-ray. Which one of the following conditions is
not part of the differential diagnosis?ia A.ALung canceria
B.APulmonary embolismia
C.AWegener's granulomatosisia
D.AChurg-Strauss syndromeia
E.ATuberculosisia

CXR: cavitating lung lesion


sqweqwesf erwrewfsdfs adasd dhe
Differential

tuberculosis
lung cancer (especially squamous cell)
abscess (Staph aureus, Klebsiella and Pseudomonas)
Wegener's granulomatosis
pulmonary embolism
rheumatoid arthritis
aspergillosis, histoplasmosis, coccidioidomycosis

Question stats

A1.2%
B34.4%
C5.1%
D58.7%
E0.6%

58.7% of users answered this question correctly


All contents of this site are ©2008 passmedicine.com - Terms and Conditions
passmedicine.compassmedicine.comReference ranges End session

Question 787 of 1500


Each one of the following predisposes to the development of obstructive sleep
apnoea, except:ia A.AAcromegalyia
B.AChronic obstructive pulmonary diseaseia
C.AAmyloidosisia
D.AObesityia
E.AHypothyroidismia

Sleep apnoea causes include obesity and macroglossia

The Sleep Heart Health Study showed that when these two conditions do coexist,
this is the result of chance alone

Obstructive sleep apnoea/hypopnoea syndrome


sqweqwesf erwrewfsdfs adasd dhe
Predisposing factors

obesity
macroglossia: acromegaly, hypothyroidism, amyloidosis
large tonsils
Marfan's syndrome

Consequence

daytime somnolence
hypertension

SIGN guidelines for the diagnosis and management of patients with OSAHS were
published in 2003

Assessment of sleepiness

Epworth Sleepiness Scale - questionnaire completed by patient +/- partner


Multiple Sleep Latency Test (MSLT) - measures the time to fall asleep in a
dark room (using EEG criteria)

Diagnostic tests

sleep studies - ranging from monitoring of pulse oximetry at night to full


polysomnography where a wide variety of physiological factors are measured
including EEG, respiratory airflow, thoraco-abdominal movement, snoring and
pulse oximetry

Management

weight loss
CPAP is first line for moderate or severe OSAHS
intra-oral devices (e.g. mandibular advancement) may be used if CPAP is not
tolerated or for patients with mild OSAHS where there is no daytime sleepiness
limited evidence to support use of pharmacological agents

Question stats

A3.8%
B47.3%
C40.8%
D1.7%
E6.4%

47.3% of users answered this question correctly


External links

BTS/SIGN
2003 OSAHS guidelines

All contents of this site are ©2008 passmedicine.com - Terms and Conditions
passmedicine.compassmedicine.comReference ranges End session

Question 787 of 1500


Each one of the following predisposes to the development of obstructive sleep
apnoea, except:ia A.AAcromegalyia
B.AChronic obstructive pulmonary diseaseia
C.AAmyloidosisia
D.AObesityia
E.AHypothyroidismia

Sleep apnoea causes include obesity and macroglossia

The Sleep Heart Health Study showed that when these two conditions do coexist,
this is the result of chance alone

Obstructive sleep apnoea/hypopnoea syndrome


sqweqwesf erwrewfsdfs adasd dhe
Predisposing factors

obesity
macroglossia: acromegaly, hypothyroidism, amyloidosis
large tonsils
Marfan's syndrome

Consequence

daytime somnolence
hypertension

SIGN guidelines for the diagnosis and management of patients with OSAHS were
published in 2003
Assessment of sleepiness

Epworth Sleepiness Scale - questionnaire completed by patient +/- partner


Multiple Sleep Latency Test (MSLT) - measures the time to fall asleep in a
dark room (using EEG criteria)

Diagnostic tests

sleep studies - ranging from monitoring of pulse oximetry at night to full


polysomnography where a wide variety of physiological factors are measured
including EEG, respiratory airflow, thoraco-abdominal movement, snoring and
pulse oximetry

Management

weight loss
CPAP is first line for moderate or severe OSAHS
intra-oral devices (e.g. mandibular advancement) may be used if CPAP is not
tolerated or for patients with mild OSAHS where there is no daytime sleepiness
limited evidence to support use of pharmacological agents

Question stats

A3.8%
B47.3%
C40.8%
D1.7%
E6.4%

47.3% of users answered this question correctly


External links

BTS/SIGN
2003 OSAHS guidelines

All contents of this site are ©2008 passmedicine.com - Terms and Conditions
passmedicine.compassmedicine.comReference ranges End session

Question 839 of 1500


A 57-year-old female presents to the Emergency Department with shortness of
breath and pleuritic chest pain. She has no past medical history of note and
enjoys good health. Investigations reveal a non-massive pulmonary embolism. What
is the recommended length of warfarinisation for this patient?ia A.A6
weeksia
B.A3 monthsia
C.A6 monthsia
D.A12 monthsia
E.ALife-longia

There are no transient risk factors for venous thromboembolism therefore the
patient should be anticoagulated for 3 months

Pulmonary embolism: management


sqweqwesf erwrewfsdfs adasd dhe
The British Thoracic Society (BTS) published guidelines in 2003 on the
management of patients with suspected pulmonary embolism (PE)

Key points from the guidelines include:

Anticoagulation

low molecular weight heparin (LMWH), rather than unfractionated heparin (UFH),
should be used routinely in patients with suspected pulmonary embolism. This
reflects the equal efficacy and safety of LMWHs as well as their ease of use.
Exceptions include patients with a massive PE or in situations where rapid
reversal of anticoagulation may be necessary
warfarinisation: the standard duration of anticoagulation is 4-6 weeks* if
temporary risk factors are present, 3 months for the first idiopathic PE and
at least 6 months for other situations

Thrombolysis

thrombolysis is now recommended as the first-line treatment for massive PE


where there is circulatory failure (e.g. hypotension). Other invasive
approaches should be considered where appropriate facilities exist

*the 2005 British Committee for Standards in Haematology guidelines suggest at


least 3 months anticoagulation in this situation, based on a 2004 study. It is
therefore possible these guidelines may soon change

Question stats

A29.2%
B44.2%
C23.7%
D1%
E1.9%

44.2% of users answered this question correctly


External links

British Thoracic Society


2003 PE guidelines

All contents of this site are ©2008 passmedicine.com - Terms and Conditions
passmedicine.compassmedicine.comReference ranges End session

Question 851 of 1500


Each one of the following is a known cause of occupational asthma, except:ia
A.AIsocyanatesia
B.ACadmiumia
C.ASoldering flux resinia
D.AFlouria
E.APlatinum saltsia

Asthma: occupational
sqweqwesf erwrewfsdfs adasd dhe
Causes

isocyanates
platinum salts
soldering flux resin
glutaraldehyde
flour
epoxy resins
proteolytic enzymes

Diagnosis

specific recommendations are made in the 2007 joint British Thoracic Society
and SIGN guidelines
serial measurements of peak expiratory flow are recommended at work and away
from work

Question stats

A16.9%
B44%
C7.5%
D16.4%
E15.2%

44% of users answered this question correctly


External links

British Thoracic Society


2007 asthma guidelines

All contents of this site are ©2008 passmedicine.com - Terms and Conditions
passmedicine.compassmedicine.comReference ranges End session

Question 856 of 1500


A preliminary diagnosis of extrinsic allergic alveolitis in a 55-year-old man.
Which one of the following features would most support this diagnosis?ia
A.AClubbingia
B.AEosinophiliaia
C.ACyanosisia
D.AFibrosis in the upper zonesia
E.AHistory of working in the steel industryia

A history of working in the steel industry and eosinophilia are not features of
extrinsic allergic alveolitis. Clubbing and cyanosis are non-specific

Extrinsic allergic alveolitis


sqweqwesf erwrewfsdfs adasd dhe
Extrinsic allergic alveolitis (EAA) is a condition caused by hypersensitivity
induced lung damage due to a variety of inhaled organic particles. It is thought
to be largely caused by immune-complex mediated tissue damage (type III
hypersensitivity) although delayed hypersensitivity (type IV) is also thought to
play a role in EAA, especially in the chronic phase

Examples

bird fanciers' lung (avian proteins)


farmers lung (spores of Micropolyspora faeni)
malt workers' lung (Aspergillus clavatus)
mushroom workers' lung (thermophilic actinomycetes*)

Presentation

acute: occur 4-8 hrs after exposure, SOB, dry cough, fever
chronic

Investigation

CXR: upper lobe fibrosis


BAL: lymphocytosis
blood: NO eosinophilia in EAA

*here the terminology is slightly confusing as thermophilic actinomycetes is an


umbrella term covering strains such as Micropolyspora faeni

Question stats

A8%
B42.6%
C6.1%
D37.4%
E5.8%

37.4% of users answered this question correctly

September 2008 exam


All contents of this site are ©2008 passmedicine.com - Terms and Conditions
passmedicine.compassmedicine.comReference ranges End session

Question 890 of 1500


Which one of the following is not part of the diagnostic criteria of acute
respiratory distress syndrome (ARDS)?ia A.ABilateral infiltrates on CXRia
B.ANon-cardiogenicia
C.ApO2/FiO2 < 200 mmHgia
D.ARespiratory rate > 24/minia
E.AAcute onsetia

ARDS
sqweqwesf erwrewfsdfs adasd dhe
Basics

acute respiratory distress syndrome


caused by increased permeability of alveolar capillaries leading to fluid
accumulation in alveoli i.e. non-cardiogenic pulmonary oedema

Criteria (American-European Consensus Conference)

1 acute onset
2 bilateral infiltrates on CXR
3 non-cardiogenic (pulmonary artery wedge pressure needed if doubt)
4pO2/FiO2 < 200 mmHg

Causes
infection: sepsis, pneumonia
massive blood transfusion
trauma
smoke inhalation
pancreatitis
cardio-pulmonary bypass

Question stats

A9.7%
B13.4%
C25%
D42.9%
E9.1%

42.9% of users answered this question correctly


All contents of this site are ©2008 passmedicine.com - Terms and Conditions
passmedicine.compassmedicine.comReference ranges End session

Question 897 of 1500


Which of the following features is associated with a good prognosis in
sarcoidosis?ia A.AInsidious onsetia
B.ASplenomegalyia
C.ADisease in black peopleia
D.AStage III features on CXRia
E.AErythema nodosumia

Erythema nodosum is associated with a good prognosis in sarcoidosis

Sarcoidosis: prognostic features


sqweqwesf erwrewfsdfs adasd dhe
Sarcoidosis is a multisystem disorder of unknown aetiology characterised by
non-caseating granulomas. It is more common in young adults and in people of
African descent. Sarcoidosis remits without treatment in approximately
two-thirds of people

Factors associated with poor prognosis

insidious onset, symptoms > 6 months


absence of erythema nodosum
extrapulmonary manifestations: e.g. lupus pernio, splenomegaly
CXR: stage III-IV features
black people

Question stats

A23.8%
B2.3%
C14.2%
D2.3%
E57.4%
57.4% of users answered this question correctly
All contents of this site are ©2008 passmedicine.com - Terms and Conditions
passmedicine.compassmedicine.comReference ranges End session

Question 898 of 1500


A 63-year-old man presents to the respiratory out-patients department with
shortness of breath. Investigations reveal a fibrosing lung disease. A sputum
sample however is positive for acid-fast bacilli. Which of the following may
have predisposed him to developing tuberculosis?ia A.ACadmiumia
B.ACoal dustia
C.AWhite asbestos fibresia
D.ABlue asbestos fibresia
E.ASilicaia

Silicosis
sqweqwesf erwrewfsdfs adasd dhe
Silicosis is a risk factor for developing TB (silica is toxic to macrophages)

Features

fibrosing lung disease


'egg-shell' calcification of hilar lymph nodes

Question stats

A7.8%
B15.9%
C7.3%
D15.2%
E53.8%

53.8% of users answered this question correctly


All contents of this site are ©2008 passmedicine.com - Terms and Conditions
passmedicine.compassmedicine.comReference ranges End session

Question 908 of 1500


Which one of the following is responsible for malt workers' lung?ia
A.AAspergillus clavatusia
B.AAvian proteinsia
C.AMycobacterium aviumia
D.AThermoActinomyces candidusia
E.AMicropolyspora faeniia

Aspergillus clavatus causes malt workers' lung, a type of EAA

Extrinsic allergic alveolitis


sqweqwesf erwrewfsdfs adasd dhe
Extrinsic allergic alveolitis (EAA) is a condition caused by hypersensitivity
induced lung damage due to a variety of inhaled organic particles. It is thought
to be largely caused by immune-complex mediated tissue damage (type III
hypersensitivity) although delayed hypersensitivity (type IV) is also thought to
play a role in EAA, especially in the chronic phase

Examples

bird fanciers' lung (avian proteins)


farmers lung (spores of Micropolyspora faeni)
malt workers' lung (Aspergillus clavatus)
mushroom workers' lung (thermophilic actinomycetes*)

Presentation

acute: occur 4-8 hrs after exposure, SOB, dry cough, fever
chronic

Investigation

CXR: upper lobe fibrosis


BAL: lymphocytosis
blood: NO eosinophilia

*here the terminology is slightly confusing as thermophilic actinomycetes is an


umbrella term covering strains such as Micropolyspora faeni

Question stats

A62.1%
B2.8%
C3.2%
D19.9%
E12%

62.1% of users answered this question correctly


All contents of this site are ©2008 passmedicine.com - Terms and Conditions
passmedicine.compassmedicine.comReference ranges End session

Question 916 of 1500


A 56-year-old man is admitted with type II respiratory failure secondary to COPD
but fails to respond to maximal medical therapy. It is decided that a trial of
non-invasive ventilation in the form of bi-level pressure support should be
given. What are the most appropriate initial settings for the ventilator?ia
A.AIPAP = 10 cm H2O; EPAP = 5 cm H2Oia
B.AIPAP = 15 cm H2O; EPAP = 15 cm H2ia
C.AIPAP = 50 cm H2O; EPAP = 20 cm H2Oia
D.AIPAP = 20 cm H2O; EPAP = 50 cm H2Oia
E.AIPAP = 5 cm H2O; EPAP = 12 cm H2Oia

The 2008 Royal College of Physicians guidelines recommend an initial IPAP of 10


cm H20. The 2002 British Thoracic Society guidelines had previously advocated
starting at 12-15 cm H20

Non-invasive ventilation
sqweqwesf erwrewfsdfs adasd dhe
The British Thoracic Society (BTS) published guidelines in 2002 on the use of
non-invasive ventilation in acute respiratory failure

Non-invasive ventilation - key indications


COPD with respiratory acidosis pH 7.25-7.35
type II respiratory failure secondary to chest wall deformity, neuromuscular
disease or obstructive sleep apnoea
cardiogenic pulmonary oedema unresponsive to CPAP
weaning from tracheal intubation

Recommended initial settings for bi-level pressure support in COPD

Expiratory Positive Airway Pressure (EPAP): 4-5 cm H2O


Inspiratory Positive Airway Pressure (IPAP): RCP advocate 10 cm H20 whilst BTS
suggest 12-15 cm H2O
back up rate: 15 breaths/min
back up inspiration:expiration ratio: 1:3

Question stats

A59.9%
B7%
C6.5%
D5.7%
E20.8%

59.9% of users answered this question correctly


External links

RCP
2008 NIV in COPD guidelines

British Thoracic Society


2002 NIV guidelines

All contents of this site are ©2008 passmedicine.com - Terms and Conditions
passmedicine.compassmedicine.comReference ranges End session

Question 968 of 1500


Which one of the following interventions is most likely to increase survival in
patients with COPD?ia A.AHome nebulisersia
B.ATiotropium inhaleria
C.ALong-term steroid therapyia
D.ASmoking cessationia
E.ALong-term oxygen therapyia

Whilst long-term oxygen therapy may increase survival in hypoxic patients,


smoking cessation is the single most important intervention in patients with
COPD

COPD: stable management


sqweqwesf erwrewfsdfs adasd dhe
General management

smoking cessation advice


pneumococcal vaccine
annual influenza vaccine

Bronchodilator therapy
short acting beta2-agoinst or anticholinergic is first-line treatment
if still symptomatic add a long-acting anti-cholinergic (e.g. tiotropium) or a
long-acting beta2-agonist (e.g. salmeterol)

Inhaled steroids

NICE recommends that an inhaled corticosteroid should for patients with an


FEV1 < 50% of predicted (i.e. moderate or severe COPD), who are having two or
more exacerbations requiring treatment with antibiotics or oral
corticosteroids in a 12 month period
reduce frequency of exacerbations

If the patient is symptomatic despite the above measures

oral theophylline

Factors which may improve survival in patients with stable COPD

smoking cessation - the single most important intervention in patients who are
still smoking
long term oxygen therapy in patients who fit criteria
lung volume reduction surgery in selected patients

Question stats

A0.2%
B0.4%
C1.9%
D51.7%
E45.8%

51.7% of users answered this question correctly

May 2005 exam


External links

NICE
COPD guidelines

Postgraduate Medical Journal


Acute COPD management

All contents of this site are ©2008 passmedicine.com - Terms and Conditions
passmedicine.compassmedicine.comReference ranges End session

Question 1015 of 1500


A 45-year-old female with a 30 pack-year history of smoking is admitted to the
Emergency Department with shortness of breath. Arterial blood gases taken on
room air are as follows:

pH7.49
pCO22.9 kPa
pO28.8 kPa

Which one of the following is the most likely diagnosis?ia A.ASalicylate


overdoseia
B.AChronic obstructive pulmonary diseaseia
C.APulmonary embolismia
D.AVomitingia
E.AAnxietyia

Pulmonary embolism needs to be excluded. Even with a significant smoking history


a reduced pO2 should not be attributed to anxiety. A salicylate overdose would
not account for a reduced pO2, unless it is severe enough to have caused
pulmonary oedema. This option is much less likely than a pulmonary embolism

Respiratory alkalosis
sqweqwesf erwrewfsdfs adasd dhe
Common causes

anxiety leading to hyperventilation


pulmonary embolism
salicylate poisoning*
CNS disorders: stroke, subarachnoid haemorrhage, encephalitis
altitude
pregnancy

*salicylate overdose leads to a mixed respiratory alkalosis and metabolic


acidosis. Early stimulation of the respiratory centre leads to a respiratory
alkalosis whilst later the direct acid effects of salicylates (combined with
acute renal failure) may lead to an acidosis

Question stats

A13.3%
B12.4%
C53.7%
D3.4%
E17.2%

53.7% of users answered this question correctly

May 2006 exam


All contents of this site are ©2008 passmedicine.com - Terms and Conditions
passmedicine.compassmedicine.comReference ranges End session

Question 1026 of 1500


A 29-year-old man with HIV is admitted with shortness of breath. He ha recently
emigrated from South Africa and has only just started taking anti-retroviral
medication. Auscultation of his chest is unremarkable although chest x-ray shows
bilateral pulmonary interstitial shadowing. What is the investigation of
choice?ia A.ABronchoalveolar lavageia
B.ACT thoraxia
C.ATransbronchial biopsyia
D.ASputum cultureia
E.ABlood cultureia

This man likely has Pneumocystis carinii pneumonia. Definitive diagnosis is by


bronchial alveolar lavage with silver staining

HIV: Pneumocystis carinii pneumonia


sqweqwesf erwrewfsdfs adasd dhe
Whilst the organism Pneumocystis carinii is now referred to as Pneumocystis
jiroveci, the term Pneumocystis carinii pneumonia (PCP) is still in common use

Pneumocystis jiroveci is an unicellular eukaryote, generally classified as a


fungus but some authorities consider it a protozoa
PCP is the most common opportunistic infection in AIDS
all patients with a CD4 count < 200/mm³ should receive PCP prophylaxis

Features

dyspnoea
dry cough
fever
very few chest signs

Extrapulmonary manifestations are rare (1-2% of cases), may cause

hepatosplenomegaly
lymphadenopathy
choroid lesions

Investigation

CXR: typically shows bilateral interstitial pulmonary infiltrates but can


present with other x-ray findings e.g. lobar consolidation. May be normal
exercise-induced desaturation
sputum often fails to show PCP, bronchoalveolar lavage (BAL) often needed to
demonstrate PCP (silver stain)

Management

co-trimoxazole
IV pentamidine in severe cases
steroids if hypoxic (if pO2 < 9.3kPa then steroids reduce risk of respiratory
failure by 50% and death by a third)

Question stats

A69.4%
B10.8%
C5.3%
D11.5%
E3%

69.4% of users answered this question correctly

September 2008 exam


All contents of this site are ©2008 passmedicine.com - Terms and Conditions
passmedicine.compassmedicine.comReference ranges End session

Question 1028 of 1500


A 65-year-old man with a history of Parkinson's disease is referred to the
respiratory clinic with shortness of breath. He has never smoked. Spirometry is
performed:

Percentage
predicted
FEV171%
FVC74%

Which one of the following drugs is most likely to be responsible?ia


A.ALevodopaia
B.AEntacaponeia
C.ARopiniroleia
D.ASelegilineia
E.APergolideia

This patient has developed pulmonary fibrosis (explaining the restrictive


picture on spirometry) secondary to pergolide therapy

Parkinson's disease: management


sqweqwesf erwrewfsdfs adasd dhe
Currently accepted practice in the management of patients with Parkinson's
disease (PD) is to delay treatment until the onset of disabling symptoms and
then to introduce a dopamine receptor agonist. If the patient is elderly,
levodopa is sometimes used as an initial treatment

1Dopamine receptor agonists


e.g. bromocriptine, ropinirole, cabergoline, apomorphine
ergot-derived dopamine receptor agonists (bromocriptine, cabergoline,
pergolide*) have been associated with pulmonary, retroperitoneal and cardiac
fibrosis. The Committee on Safety of Medicines advice that an ESR, creatinine
and chest x-ray should be obtained prior to treatment and patients should be
closely monitored

2Levodopa
usually combined with a decarboxylase inhibitor (e.g. carbidopa or
benserazide) to prevent peripheral metabolism of levodopa to dopamine
reduced effectiveness with time (usually by 2 years)
unwanted effects: dyskinesia, 'on-off' effect
no use in neuroleptic induced parkinsonism

3MAO-B (Monoamine Oxidase-B) inhibitors


e.g. selegiline
inhibits the breakdown of dopamine secreted by the dopaminergic neurons

4Amantadine
mechanism is not fully understood, probably increases dopamine release and
inhibits its uptake at dopaminergic synapses

5COMT (Catechol-O-Methyl Transferase) inhibitors


e.g. entacapone
COMT is an enzyme involved in the breakdown of dopamine, and hence may be used
as an adjunct to levodopa therapy
used in established PD
6Antimuscarinics
block cholinergic receptors
now used more to treat drug-induced parkinsonism rather than idiopathic
Parksinson's disease
help tremor and rigidity
e.g. procyclidine, benzotropine, trihexyphenidyle (benzhexol)

*pergolide was withdrawn from the US market in March 2007 due to concern
regarding increased incidence of valvular dysfunction

Question stats

A10.9%
B7.5%
C15.1%
D19.6%
E47%

47% of users answered this question correctly

May 2008 exam


External links

NICE
2006 Parkinson's disease guidelines

Postgraduate medical journals


Levodopa complications

All contents of this site are ©2008 passmedicine.com - Terms and Conditions
passmedicine.compassmedicine.comReference ranges End session

Question 1036 of 1500


A 54-year-old woman with a 30-pack-year history of smoking presents due to
increasing breathlessness. A diagnosis of chronic obstructive pulmonary disease
(COPD) is suspected. Which of the following diagnostic criteria should be used
when assessing a patient with suspected COPD?ia A.AFEV1 > 70% of
predicted value + FEV1/FVC < 60%ia
B.AFEV1 < 80% of predicted value + FEV1/FVC < 70%ia
C.AFEV1 < 70% of predicted value + FEV1/FVC < 70%ia
D.AFEV1 < 80% of predicted value + FEV1/FVC < 60%ia
E.AFEV1 < 70% of predicted value + FEV1/FVC > 70%ia

COPD: investigation and diagnosis


sqweqwesf erwrewfsdfs adasd dhe
The following investigations are recommended in patients with suspected COPD:

spirometry to demonstrate airflow obstruction: forced expiratory volume in 1


second (FEV1) less than 80% of the predicted value and FEV1/FVC ratio less
than 70%
chest x-ray: hyperinflation, bullae, flat hemidiaphragm. Also important to
exclude lung cancer
full blood count: exclude secondary polycythaemia
Measuring peak expiratory flow is of limited value in COPD, as it may
underestimate the degree of airflow obstruction

Question stats

A4.4%
B46.3%
C31%
D10%
E8.3%

46.3% of users answered this question correctly


All contents of this site are ©2008 passmedicine.com - Terms and Conditions
passmedicine.compassmedicine.comReference ranges End session

Question 1081 of 1500


A 19-year-old male with no past medical history presents to the Emergency
Department with anterior chest pain and shortness of breath. Blood pressure is
110/80 mmHg and his pulse is 84 bpm. The chest x-ray is reported as showing a
50% pneumothorax with no mid-line shift. What is the most appropriate
management?ia A.AIntercostal drain insertionia
B.AImmediate 14G cannula into 2nd intercostal space, mid-clavicular
lineia
C.ADischargeia
D.AAdmit for 48 hours observation and repeat chest x-rayia
E.AAspirationia

Pneumothorax
sqweqwesf erwrewfsdfs adasd dhe
The British Thoracic Society (BTS) published guidelines for the management of
spontaneous pneumothorax in 2003. A pneumothorax is termed primary if there is
no underlying lung disease and secondary if there is

Primary pneumothorax

Recommendations include:

if the rim of air is < 2cm and the patient is not short of breath then
discharge should be considered
otherwise aspiration should be attempted
if this fails then repeat aspiration should be considered
if this fails then a chest drain should be inserted

Secondary pneumothorax

Recommendations include:

if the patient is > 50 years old and the rim of air is > 2cm and the patient
is short of breath then a chest drain should be inserted.
otherwise aspiration should be attempted. If aspiration fails a chest drain
should be inserted. All patients should be admitted for at least 24 hours

Iatrogenic pneumothorax
Recommendations include:

less likelihood of recurrence than spontaneous pneumothorax


majority will resolve with observation, if treatment is required then
aspiration should be used
ventilated patients need chest drains, as may some patients with COPD

Question stats

A23.1%
B10.4%
C2.6%
D2.8%
E61.1%

61.1% of users answered this question correctly


External links

British Thoracic Society


Pneumothorax guidelines

All contents of this site are ©2008 passmedicine.com - Terms and Conditions
passmedicine.compassmedicine.comReference ranges End session

Question 1086 of 1500


Which one of the following is the most common type of lung cancer in the UK?ia
A.ASmall cellia
B.ASquamous cellia
C.AAdenocarcinomaia
D.ALarge cellia
E.ACarcinoidia

Tricky question. It is well known that the incidence of adenocarcinoma is rising


in comparison to the other types of non-small cell lung cancer. Indeed,
adenocarcinoma is now the most common type of lung cancer in the USA. In the UK
however squamous cell cancer remains the most common subtype

Reference

Janssen-Heijnen, M.L. and J.W. Coebergh, The changing epidemiology of lung


cancer in Europe. Lung Cancer, 2003. 41(3)

Lung cancer: types


sqweqwesf erwrewfsdfs adasd dhe
Lung cancer

squamous: c. 35%
adenocarcinoma: c. 30%
small (oat) cell: c. 15%
large cell: c. 10%
other c. 5%

Other tumours

alveolar cell carcinoma: not related to smoking, ++sputum


bronchial adenoma: mostly carcinoid
Question stats

A13.7%
B57.3%
C26.1%
D2.3%
E0.6%

57.3% of users answered this question correctly


All contents of this site are ©2008 passmedicine.com - Terms and Conditions
passmedicine.compassmedicine.comReference ranges End session

Question 1094 of 1500


A 54-year-old woman with chronic obstructive pulmonary disease (COPD) is
prescribed an inhaled corticosteroid. What is the main therapeutic benefit of
inhaled corticosteroids in patients with COPD?ia A.AReduced severity of
exacerbationsia
B.AImproved all cause mortalityia
C.AReduced use of bronchodilatorsia
D.ASlows decline in FEV1ia
E.AReduced frequency of exacerbationsia

COPD - reason for using inhaled corticosteroids - reduced exacerbations

COPD: stable management


sqweqwesf erwrewfsdfs adasd dhe
General management

smoking cessation advice


pneumococcal vaccine
annual influenza vaccine

Bronchodilator therapy

short acting beta2-agoinst or anticholinergic is first-line treatment


if still symptomatic add a long-acting anti-cholinergic (e.g. tiotropium) or a
long-acting beta2-agonist (e.g. salmeterol)

Inhaled steroids

NICE recommends that an inhaled corticosteroid should for patients with an


FEV1 < 50% of predicted (i.e. moderate or severe COPD), who are having two or
more exacerbations requiring treatment with antibiotics or oral
corticosteroids in a 12 month period

reduce frequency of exacerbations


If the patient is symptomatic despite the above measures

oral theophylline

Factors which may improve survival in patients with stable COPD


smoking cessation - the single most important intervention in patients who are
still smoking
long term oxygen therapy in patients who fit criteria
lung volume reduction surgery in selected patients

Question stats

A12.4%
B2.6%
C4.4%
D5.2%
E75.4%

75.4% of users answered this question correctly

May 2008 exam


External links

NICE
COPD guidelines

Postgraduate Medical Journal


Acute COPD management

All contents of this site are ©2008 passmedicine.com - Terms and Conditions
passmedicine.compassmedicine.comReference ranges End session

Question 1116 of 1500


Which one of the following is associated with a poor prognosis in patients with
community-acquired pneumonia?ia A.ADiastolic blood pressure 65 mmHgia
B.ASodium 131 mmol/lia
C.AUrea 12 mmol/lia
D.AWhite blood cell 27 * 109/lia
E.ARespiratory rate 25/minia

Pneumonia: prognostic factors


sqweqwesf erwrewfsdfs adasd dhe
CURB-65 criteria of severe pneumonia

Confusion (abbreviated mental test score < 8/10)


Urea > 7 mmol/L
Respiratory rate >= 30 / min
BP: systolic < 90 or diastolic < 60 mmHg
age > 65 years

Patients with 3 or more (out of 5) of the above criteria are regarded as having
a severe pneumonia

Other factors associated with a poor prognosis include:

presence of coexisting disease


hypoxaemia (pO2 < 8 kPa) independent of FiO2
Question stats

A4.9%
B2.8%
C80.6%
D5.5%
E6.2%

80.6% of users answered this question correctly

January 2006 exam


External links

British Thoracic Society


2004 CAP guidelines update

All contents of this site are ©2008 passmedicine.com - Terms and Conditions
passmedicine.compassmedicine.comReference ranges End session

Question 1127 of 1500


A newborn female baby is diagnosed with cystic fibrosis following an episode of
meconium ileus shortly after birth. Which one of the following is least likely
to occur as a consequence?ia A.ADelayed pubertyia
B.ANasal polypsia
C.ADiabetes mellitusia
D.ARectal prolapseia
E.AArthropathyia

Arthropathy is not a common feature of cystic fibrosis

Cystic fibrosis: features


sqweqwesf erwrewfsdfs adasd dhe
Presenting features

neonatal period (around 20%): meconium ileus, less commonly prolonged jaundice
recurrent chest infections (40%)
malabsorption (30%): steatorrhoea, failure to thrive
other features (10%): liver disease

Other features of cystic fibrosis

short stature
diabetes mellitus
delayed puberty
rectal prolapse (due to bulky stools)
nasal polyps
male infertility, female subfertility

Question stats

A13.5%
B11.1%
C9.4%
D20.1%
E45.9%

45.9% of users answered this question correctly


All contents of this site are ©2008 passmedicine.com - Terms and Conditions
passmedicine.compassmedicine.comReference ranges End session

Question 1183 of 1500


A 38-year-old man is reviewed in the respiratory clinic complaining of episodic
wheezing whilst playing rugby. There is no history of cough, atopy or smoking.
He is generally fit and well and has no past medical history of note. Clinical
examination is unremarkable. Following history and examination it is thought he
has an intermediate probability of asthma. Which one of the following is the
most appropriate next investigation?ia A.ASpirometryia
B.ASerial peak expiratory flow measurementsia
C.AHistamine stimulation testia
D.AMethacholine stimulation testia
E.AA trial of inhaled steroids with FEV1 measurements before and afteria

Asthma - intermediate probability - do spirometry first-line

If the FEV1/FVC < 0.7 then a trial of treatment is appropriate. Otherwise


further investigations should be performed

Asthma: diagnosis in adults


sqweqwesf erwrewfsdfs adasd dhe
The 2008 British Thoracic Society guidelines marked a subtle change in the
approach to diagnosing asthma. It suggests dividing patients into a high,
intermediate and low probability of having asthma based on the presence or
absence of typical symptoms. A list can be found in the external link but
include typical symptoms such as wheeze, nocturnal cough etc

Example of features used to assess asthma (not complete, please see link)

Increase possibility of asthma Decrease possibility of asthma


• Wheeze, breathlessness, chest tightness and cough, worse at night/early
morning
• History of atopic disorder
• Wheeze heard on auscultation

• Unexplained peripheral blood eosinophilia• Prominent dizziness,


light-headedness, peripheral tingling
• Chronic productive cough in the absence of wheeze or breathlessness
• Repeatedly normal physical examination
• Significant smoking history (i.e. > 20 pack-years)
• Normal PEF or spirometry when symptomatic

Management is based on this assessment:

high probability: trial of treatment


intermediate probability: see below
low probability: investigate/treat other condition

For patients with an intermediate probability of asthma further investigations


are suggested. The guidelines state that spirometry is the preferred initial
test:

FEV1/FVC < 0.7: trial of treatment


FEV1/FVC > 0.7: further investigation/consider referral
Recent studies have shown the limited value of other 'objective' tests. It is
now recognised that in patients with normal or near-normal pre-treatment lung
function there is little room for measurable improvement in FEV1 or peak flow.

A > 400 ml improvement in FEV1 is considered significant

before and after 400 mcg inhaled salbutamol in patients with diagnostic
uncertainty and airflow obstruction present at the time of assessment
if there is an incomplete response to inhaled salbutamol, after either inhaled
corticosteroids (200 mcg twice daily beclometasone equivalent for 6-8 weeks)
or oral prednisolone (30 mg once daily for 14 days)

It is now advised to interpret peak flow variability with caution due to the
poor sensitivity of the test

diurnal variation % = [(Highest – Lowest PEFR) / Highest PEFR] x 100


assessment should be made over 2 weeks
greater than 20% diurnal variation is considered significant

Question stats

A35.9%
B34%
C1.3%
D5.3%
E23.6%

35.9% of users answered this question correctly


External links

British Thoracic Society


2008 Asthma guidelines

All contents of this site are ©2008 passmedicine.com - Terms and Conditions
passmedicine.compassmedicine.comReference ranges End session

Question 1201 of 1500


A 23-year-old female who is 28 weeks pregnant presents with shortness-of-breath
and right-sided pleuritic chest pain. A diagnosis of pulmonary embolism is
suspected. Which one of the following statements regarding the appropriate
management is incorrect?ia A.AChest x-ray should be performedia
B.APositive compression duplex Doppler may negate the need for further
investigationia
C.AVentilation-perfusion scanning exposes the fetus to less radiation than
computed tomographic pulmonary angiographyia
D.AD-dimer levels are of no useia
E.AComputed tomographic pulmonary angiography increases the lifetime risk
of breast cancer in the pregnant womenia

Pregnancy: DVT/PE investigation


sqweqwesf erwrewfsdfs adasd dhe
Guidelines were published in 2007 by the Royal College of Obstetricians. Key
points include:
chest x-ray should be performed in all patients
- compression duplex Doppler should be performed if the chest x-ray is normal -
this may provide indirect evidence of a pulmonary embolism and negate the need
for further radiation exposure

the decision to perform a V/Q or CTPA should be taken at a local level after
discussion with the patient and radiologist
CTPA exposes the fetus to about 10-30% of the radiation dose of a V/Q scan
V/Q scanning exposes the maternal breast tissue to less radiation than a CTPA

D-dimer is of no use in the investigation of thromboembolism as it raised in


pregnancy

Question stats

A14.5%
B12.5%
C37.4%
D12.6%
E23%

37.4% of users answered this question correctly

January 2006 exam


External links

Royal College of Obstetricians and Gynaecologists


Thromboembolic disease in pregnancy and the puerperium: acute management

All contents of this site are ©2008 passmedicine.com - Terms and Conditions
passmedicine.compassmedicine.comReference ranges End session

Question 1209 of 1500


A 52-year-old male is admitted to hospital with a temperature of 38.2 C and a 3
days history of a productive cough. He has been generally unwell for the past 10
days with flu-like symptoms. Chest x-ray shows left lower zone consolidation.
What is the likely causative organism?ia A.AMoraxella catarrhalisia
B.AFusobacteriumia
C.AKlebsiellaia
D.AStaphylococcus aureusia
E.AChlamydia pneumoniaeia

Preceding influenza predisposes to Staphylococcus aureus


pneumonia
Pneumonia: community-acquired
sqweqwesf erwrewfsdfs adasd dhe
Community acquired pneumonia (CAP) may be caused by the following organisms:

Streptococcus pneumoniae (accounts for around 80% of cases)


Haemophilus influenzae
Staphylococcal aureus
atypical pneumonias (e.g. due to Mycoplasma pneumoniae)
viruses
Streptococcus pneumoniae (pneumococcus) is the most common cause of
community-acquired pneumonia

Characteristic features of pneumococcal pneumonia

rapid onset
high fever
pleuritic chest pain
herpes labialis

Antibiotic choices

home-treated uncomplicated CAP: first line - oral amoxicillin


hospitalized uncomplicated CAP: if admitted for non-clinical reasons or not
previously treated in the community then oral amoxicillin, otherwise
amoxicillin + macrolide

Question stats

A20.8%
B0.7%
C14.4%
D52.4%
E11.7%

52.4% of users answered this question correctly

September 2008 exam


External links

British Thoracic Society


2004 CAP guidelines update

All contents of this site are ©2008 passmedicine.com - Terms and Conditions
passmedicine.compassmedicine.comReference ranges End session

Question 1218 of 1500


What is the first line treatment in allergic bronchopulmonary aspergillosis?ia
A.AItraconazoleia
B.ANebulised pentamidineia
C.AFluconazoleia
D.ACyclophosphamideia
E.APrednisoloneia

Allergic bronchopulmonary aspergillosis


sqweqwesf erwrewfsdfs adasd dhe
Allergic bronchopulmonary aspergillosis results from an allergy to Aspergillus
spores. In the exam questions often give a history of bronchiectasis and
eosinophilia.
Features

bronchoconstriction: wheeze, cough, dyspnoea


bronchiectasis (proximal)

Investigations

eosinophilia
flitting CXR changes
positive radioallergosorbent (RAST) test to Aspergillus
positive IgG precipitins (not as positive as in aspergilloma)
raised IgE

Management

steroids
itraconazole is sometimes introduced as a second line agent

Question stats

A19.2%
B8.6%
C12.3%
D2.8%
E57%

57% of users answered this question correctly


All contents of this site are ©2008 passmedicine.com - Terms and Conditions
passmedicine.compassmedicine.comReference ranges End session

Question 1260 of 1500


A 34-year-old steelworker presents complaining of episodic shortness of breath.
This is particularly noted whilst at work where he describes feeling wheezy and
having a tendency to cough. Which one of the following is the most appropriate
diagnostic investigation?ia A.APatch testingia
B.AHigh resolution computed tomography of thoraxia
C.ASerial peak flow measurements at work and at homeia
D.ASpecific IgE measurementsia
E.ASkin prick testia

Asthma: occupational
sqweqwesf erwrewfsdfs adasd dhe
Causes

isocyanates
platinum salts
soldering flux resin
glutaraldehyde
flour
epoxy resins
proteolytic enzymes

Diagnosis

specific recommendations are made in the 2007 joint British Thoracic Society
and SIGN guidelines
serial measurements of peak expiratory flow are recommended at work and away
from work

Question stats

A4.3%
B4.9%
C74.7%
D7.7%
E8.4%

74.7% of users answered this question correctly

January 2008 exam


External links

British Thoracic Society


2007 asthma guidelines

All contents of this site are ©2008 passmedicine.com - Terms and Conditions
passmedicine.compassmedicine.comReference ranges End session

Question 1302 of 1500


A chest x-ray of a patient with sarcoidosis shows bilateral hilar
lymphadenopathy accompanied with interstitial infiltrates. What chest x-ray
stage does this correspond to?ia A.AStage 0ia
B.AStage 1ia
C.AStage 2ia
D.AStage 3ia
E.AStage 4ia

Sarcoidosis CXR

1 = BHL
2 = BHL + infiltrates
3 = infiltrates
4 = fibrosis

Sarcoidosis: investigation
sqweqwesf erwrewfsdfs adasd dhe
There is no one diagnostic test for sarcoidosis and hence diagnosis is still
largely clinical. ACE levels have a sensitivity of 60% and specificity of 70%
and are therefore not reliable in the diagnosis of sarcoidosis although they may
have a role in monitoring disease activity. Routine bloods may show
hypercalcaemia (seen in 10% if patients) and a raised ESR
A chest x-ray may show the following changes:

stage 0 = normal
stage 1 = bilateral hilar lymphadenopathy (BHL)
stage 2 = BHL + interstitial infiltrates
stage 3 = diffuse interstitial infiltrates only
stage 4 = diffuse fibrosis

Other investigations*

spirometry: may show a restrictive defect


tissue biopsy: non-caseating granulomas
gallium-67 scan - not used routinely

*the Kveim test (where part of the spleen from a patient with known sarcoidosis
is injected under the skin) is no longer performed due to concerns about
cross-infection

Question stats

A0.5%
B3.6%
C74.5%
D18.7%
E2.7%

74.5% of users answered this question correctly


All contents of this site are ©2008 passmedicine.com - Terms and Conditions
passmedicine.compassmedicine.comReference ranges End session

Question 1317 of 1500


A 62-year-old woman with recently diagnosed chronic obstructive pulmonary
disease (COPD) presents for review. Her FEV1 is 65% of the predicted value. She
has managed to give up smoking and was prescribed a salbutamol inhaler to use as
required. Despite this she is still symptomatic and complains of wheeze and
shortness of breath. What is the most appropriate next step?ia A.AAdd
inhaled corticosteroidia
B.AAdd long-acting anti-cholinergic inhaleria
C.ARefer for consideration of long-term oxygen therapyia
D.AAdd oral theophyllineia
E.AAdd combination long-acting beta2-agonist and corticosteroid inhaleria

Current NICE guidelines recommend adding a combined short-acting beta2-agonist


and anti-cholinergic (e.g. Combivent) in this situation. This has however
recently been withdrawn. Until new guidelines are issued adding a long-acting
anti-cholinergic inhaler seems a reasonable next step. This patient does not yet
meet the criteria for an inhaled corticosteroid

COPD: stable management


sqweqwesf erwrewfsdfs adasd dhe
General management

smoking cessation advice


pneumococcal vaccine
annual influenza vaccine
Bronchodilator therapy

short acting beta2-agoinst or anticholinergic is first-line treatment


if still symptomatic add a long-acting anti-cholinergic (e.g. tiotropium) or a
long-acting beta2-agonist (e.g. salmeterol)

Inhaled steroids

NICE recommends that an inhaled corticosteroid should for patients with an


FEV1 < 50% of predicted (i.e. moderate or severe COPD), who are having two or
more exacerbations requiring treatment with antibiotics or oral
corticosteroids in a 12 month period
reduce frequency of exacerbations

If the patient is symptomatic despite the above measures

oral theophylline

Factors which may improve survival in patients with stable COPD

smoking cessation - the single most important intervention in patients who are
still smoking
long term oxygen therapy in patients who fit criteria
lung volume reduction surgery in selected patients

Question stats

A15.7%
B60.4%
C2.5%
D1.8%
E19.6%

60.4% of users answered this question correctly


External links

NICE
COPD guidelines

Postgraduate Medical Journal


Acute COPD management

All contents of this site are ©2008 passmedicine.com - Terms and Conditions
passmedicine.compassmedicine.comReference ranges End session

Question 1322 of 1500


A 35-year-old female presents with shortness of breath. The following blood
gases are obtained on room air:

pH7.54
pCO21.8 kPa
pO212.4 kPa

Which one of the following is the least likely cause?ia A.AOpiate


overdoseia
B.APulmonary embolismia
C.APregnancyia
D.AEncephalitisia
E.AAnxietyia

The question asks for the least likely cause of a respiratory alkalosis.
Salicylate, not opiate, poisoning is associated with a respiratory alkalosis.
Opiate overdose would lead to respiratory depression and hence a respiratory
acidosis

Respiratory alkalosis
sqweqwesf erwrewfsdfs adasd dhe
Common causes

anxiety leading to hyperventilation


pulmonary embolism
salicylate poisoning*
CNS disorders: stroke, subarachnoid haemorrhage, encephalitis
altitude
pregnancy

*salicylate overdose leads to a mixed respiratory alkalosis and metabolic


acidosis. Early stimulation of the respiratory centre leads to a respiratory
alkalosis whilst later the direct acid effects of salicylates (combined with
acute renal failure) may lead to an acidosis

Question stats

A44.6%
B14.5%
C5.4%
D3.3%
E32.1%

44.6% of users answered this question correctly


All contents of this site are ©2008 passmedicine.com - Terms and Conditions
passmedicine.compassmedicine.comReference ranges End session

Question 1356 of 1500


A 57-year-old man with a history of COPD is admitted to hospital with worsening
shortness-of-breath. He started a course of co-amoxiclav from his GP 5 days ago.
On examination blood pressure is 124/88 mmHg with a respiratory rate of 18 /
min. A chest x-ray reveals left lower lobe consolidation. Arterial blood gases
on air are as follows:

pH7.37
pCO25.5 kPa
pO29.1 kPa

What is the most suitable antibiotic therapy?ia A.AOral cefacloria


B.AOral levofloxacinia
C.AOral amoxicillin + erythromycinia
D.AIV ceftriaxone + clarithromycinia
E.AOral co-amoxiclav + metronidazoleia

The 2004 British Thoracic Society pneumonia guidelines do not make specific
recommendations for patients with COPD. Whilst COPD may obviously affect the
severity of the episode there is limited evidence to suggest that the causative
organisms are different. Oral amoxicillin with an oral macrolide is therefore
first line treatment for hospitalised patients with non-severe CAP

Pneumonia: community-acquired
sqweqwesf erwrewfsdfs adasd dhe
Community acquired pneumonia (CAP) may be caused by the following organisms:

Streptococcus pneumoniae (accounts for around 80% of cases)


Haemophilus influenzae
Staphylococcal aureus
atypical pneumonias (e.g. due to Mycoplasma pneumoniae)
viruses

Streptococcus pneumoniae (pneumococcus) is the most common cause of


community-acquired pneumonia

Characteristic features of pneumococcal pneumonia

rapid onset
high fever
pleuritic chest pain
herpes labialis

Antibiotic choices

home-treated uncomplicated CAP: first line - oral amoxicillin


hospitalized uncomplicated CAP: if admitted for non-clinical reasons or not
previously treated in the community then oral amoxicillin, otherwise
amoxicillin + macrolide

Question stats

A2.5%
B7.1%
C64.5%
D22.4%
E3.4%

64.5% of users answered this question correctly

January 2006 exam


External links

British Thoracic Society


2004 CAP guidelines update

All contents of this site are ©2008 passmedicine.com - Terms and Conditions
passmedicine.compassmedicine.comReference ranges End session

Question 1362 of 1500


A 35-year-old patient with a history of asthma and epilepsy presents with
haemoptysis and a worsening of his asthma. Blood tests reveal an eosinophilia
and a positive pANCA. Which of the following drugs is most likely to have
precipitated the likely diagnosis?ia A.AMontelukastia
B.APhenytoinia
C.ASodium valproateia
D.APrednisoloneia
E.ASodium cromoglycateia

This patient probably has Churg-Strauss syndrome, which is associated with the
use of leukotriene receptor antagonists

Churg-Strauss syndrome
sqweqwesf erwrewfsdfs adasd dhe
Churg-Strauss syndrome is an ANCA associated small-medium vessel vasculitis

Features

asthma
blood eosinophilia (e.g. > 10%)
paranasal sinusitis
mononeuritis multiplex
pANCA positive in 60%

Leukotriene receptor antagonists may precipitate the disease

Question stats

A41.3%
B25.2%
C18.1%
D5.4%
E9.9%

41.3% of users answered this question correctly


All contents of this site are ©2008 passmedicine.com - Terms and Conditions
passmedicine.compassmedicine.comReference ranges End session

Question 1369 of 1500


A 60-year-old female with a history of COPD presents to the Emergency Department
with shortness of breath. Blood pressure is 120/80 mmHg and he pulse is 90 bpm.
The chest x-ray shows a pneumothorax with a 2.5 cm rim of air and no mediastinal
shift. What is the most appropriate management?ia A.AIntercostal drain
insertionia
B.ADischargeia
C.AAdmit for 48 hours observation and repeat chest x-rayia
D.AImmediate 14G cannula into 2nd intercostal space, mid-clavicular
lineia
E.AAspirationia

Pneumothorax
sqweqwesf erwrewfsdfs adasd dhe
The British Thoracic Society (BTS) published guidelines for the management of
spontaneous pneumothorax in 2003. A pneumothorax is termed primary if there is
no underlying lung disease and secondary if there is

Primary pneumothorax

Recommendations include:
if the rim of air is < 2cm and the patient is not short of breath then
discharge should be considered
otherwise aspiration should be attempted
if this fails then repeat aspiration should be considered
if this fails then a chest drain should be inserted

Secondary pneumothorax

Recommendations include:

if the patient is > 50 years old and the rim of air is > 2cm and the patient
is short of breath then a chest drain should be inserted.
otherwise aspiration should be attempted. If aspiration fails a chest drain
should be inserted. All patients should be admitted for at least 24 hours

Iatrogenic pneumothorax

Recommendations include:

less likelihood of recurrence than spontaneous pneumothorax


majority will resolve with observation, if treatment is required then
aspiration should be used
ventilated patients need chest drains, as may some patients with COPD

Question stats

A49.3%
B1.7%
C3%
D2.3%
E43.7%

49.3% of users answered this question correctly


External links

British Thoracic Society


Pneumothorax guidelines

All contents of this site are ©2008 passmedicine.com - Terms and Conditions
passmedicine.compassmedicine.comReference ranges End session

Question 1384 of 1500


A 17-year-old male with a history of cystic fibrosis presents to clinic for
annual review. What is the most appropriate advice regarding his diet?ia
A.AHigh calorie and low fat with pancreatic enzyme supplementation for
every mealia
B.AHigh calorie and low fat with pancreatic enzyme supplementation for
evening mealia
C.ANormal calorie and low fat with pancreatic enzyme supplementation for
every mealia
D.AHigh calorie and high fat with pancreatic enzyme supplementation for
evening mealia
E.AHigh calorie and high fat with pancreatic enzyme supplementation for
every mealia
Cystic fibrosis: management
sqweqwesf erwrewfsdfs adasd dhe
Management of cystic fibrosis involves a multidisciplinary approach

Key points

regular (at least twice daily) chest physiotherapy and postural drainage.
Parents are usually taught to do this. Deep breathing exercises are also
useful
high calorie diet, including high fat intake*
vitamin supplementation
pancreatic enzyme supplements taken with meals
heart and lung transplant

*this is now the standard recommendation - previously high calorie, low-fat


diets have been recommended to reduce the amount of steatorrhoea

Question stats

A28.1%
B10.6%
C14.9%
D6.9%
E39.5%

39.5% of users answered this question correctly


All contents of this site are ©2008 passmedicine.com - Terms and Conditions
passmedicine.compassmedicine.comReference ranges End session

Question 1390 of 1500


A 25-year-old man is referred due to pain and swelling in his knees and ankles.
On examination he has a painful, erythematous rash on his legs. The following
results are obtained:

Rheumatoid factorNegative
ESR94 mm/hr
Chest x-rayHilar lymphadenopathy

What is the most likely outcome?ia A.AImprovement following a course of


prednisoloneia
B.AScarring and ulceration of skinia
C.ASpontaneous improvementia
D.AProgressive arthritisia
E.ARenal replacement therapy in 20 years timeia

The majority of patients with sarcoidosis get better without treatment

This man has an acute form of sarcoidosis. There are no indications for steroid
therapy and his symptoms will resolve spontaneously in the majority of cases

Sarcoidosis: prognostic features


sqweqwesf erwrewfsdfs adasd dhe
Sarcoidosis is a multisystem disorder of unknown aetiology characterised by
non-caseating granulomas. It is more common in young adults and in people of
African descent. Sarcoidosis remits without treatment in approximately
two-thirds of people

Factors associated with poor prognosis

insidious onset, symptoms > 6 months


absence of erythema nodosum
extrapulmonary manifestations: e.g. lupus pernio, splenomegaly
CXR: stage III-IV features
black people

Question stats

A30.1%
B1.2%
C64.3%
D1.9%
E2.5%

64.3% of users answered this question correctly

September 2006 exam


All contents of this site are ©2008 passmedicine.com - Terms and Conditions
passmedicine.compassmedicine.comReference ranges End session

Question 1397 of 1500


A 49-year-old female is admitted to the Emergency Department with shortness of
breath. On examination the pulse is 114 bpm with blood pressure 106/66 mmHg,
temperature 37.7ºC and respiratory rate 30/min. Examination of the
cardiorespiratory system is unremarkable with a peak expiratory flow rate of 400
l/min. Arterial blood gases on air reveal:

pH7.41
pCO24.0 kPa
pO27.2 kPa

Following the initiation of oxygen therapy, what is the next most important step
in management?ia A.AIV aminophyllineia
B.AIV hydrocortisoneia
C.ALow molecular weight heparinia
D.AIV fluidsia
E.AIV co-trimoxazoleia

Type 1 respiratory failure in a tachycardic, tachypnoeic female with an absence


of chest signs points towards a diagnosis of pulmonary embolism

Pulmonary embolism: management


sqweqwesf erwrewfsdfs adasd dhe
The British Thoracic Society (BTS) published guidelines in 2003 on the
management of patients with suspected pulmonary embolism (PE)

Key points from the guidelines include:

Anticoagulation

low molecular weight heparin (LMWH), rather than unfractionated heparin (UFH),
should be used routinely in patients with suspected pulmonary embolism. This
reflects the equal efficacy and safety of LMWHs as well as their ease of use.
Exceptions include patients with a massive PE or in situations where rapid
reversal of anticoagulation may be necessary
warfarinisation: the standard duration of anticoagulation is 4-6 weeks* if
temporary risk factors are present, 3 months for the first idiopathic PE and
at least 6 months for other situations

Thrombolysis

thrombolysis is now recommended as the first-line treatment for massive PE


where there is circulatory failure (e.g. hypotension). Other invasive
approaches should be considered where appropriate facilities exist

*the 2005 British Committee for Standards in Haematology guidelines suggest at


least 3 months anticoagulation in this situation, based on a 2004 study. It is
therefore possible these guidelines may soon change

Question stats

A2.7%
B12.7%
C66.3%
D15.1%
E3.1%

66.3% of users answered this question correctly


External links

British Thoracic Society


2003 PE guidelines

All contents of this site are ©2008 passmedicine.com - Terms and Conditions
passmedicine.compassmedicine.comReference ranges End session

Question 1398 of 1500


A 63-year-old man is noted to have a pleural effusion on CXR. Which one of the
following would typically cause a transudate?ia A.APancreatitisia
B.APneumoniaia
C.AYellow nail syndromeia
D.AHypothyroidismia
E.ADressler's syndromeia

Pleural effusion
sqweqwesf erwrewfsdfs adasd dhe
Exudate (> 30g/L protein)

infection: pneumonia, TB, subphrenic abscess


connective tissue disease: RA, SLE
neoplasia: lung cancer, mesothelioma, metastases
pancreatitis
pulmonary embolism
Dressler's syndrome
yellow nail syndrome

Transudate (< 30g/L protein)


heart failure
hypoalbuminaemia (liver disease, nephrotic syndrome, malabsorption)

hypothyroidism
Meigs' syndrome

Question stats

A9.2%
B6.6%
C9.8%
D56.2%
E18.1%

56.2% of users answered this question correctly


All contents of this site are ©2008 passmedicine.com - Terms and Conditions
passmedicine.compassmedicine.comReference ranges End session

Question 34 of 106
A 48-year-old salesman presents with a 5 day history of cough and
pleuritic chest pain. He has no past medical history of note. On examination his
temperature is 38.2ºC, blood pressure is 120/80 mmHg, respiratory rate 18/min
and pulse 84/min. Auscultation of the chest reveals decreased air entry in the
left base and the same area is dull to percussion. What is the most suitable
management?ia A.AOral amoxicillinia
B.AOral co-amoxiclavia
C.AOral amoxicillin + erythromycinia
D.AOral erythromycinia
E.AAdmitia

Pneumonia: community-acquired
sqweqwesf erwrewfsdfs adasd dhe
Community acquired pneumonia (CAP) may be caused by the following organisms:

Streptococcus pneumoniae (accounts for around 80% of cases)


Haemophilus influenzae
Staphylococcal aureus
atypical pneumonias (e.g. due to Mycoplasma pneumoniae)
viruses

Streptococcus pneumoniae (pneumococcus) is the most common cause of


community-acquired pneumonia

Characteristic features of pneumococcal pneumonia

rapid onset
high fever
pleuritic chest pain
herpes labialis

Antibiotic choices
home-treated uncomplicated CAP: first line - oral amoxicillin
hospitalized uncomplicated CAP: if admitted for non-clinical reasons or not
previously treated in the community for this episode then oral amoxicillin,
otherwise amoxicillin + macrolide

Question stats

A53.8%
B9.1%
C23.4%
D3.5%
E10.1%

53.8% of users answered this question correctly

Session score = 52.9% External links

British Thoracic Society


2004 CAP guidelines update

All contents of this site are ©2009 passmedicine.com - Terms and Conditions
passmedicine.compassmedicine.comReference ranges End session

Question 32 of 60
Which one of the following is least associated with a false positive sweat test
in the diagnosis of cystic fibrosisia A.ADiabetes mellitusia
B.AHypothyroidismia
C.AG6PDia
D.AMalnutritionia
E.AEctodermal dysplasiaia

Cystic fibrosis: diagnosis


sqweqwesf erwrewfsdfs adasd dhe
Sweat test

patient's with CF have abnormally high sweat chloride


normal value < 40 mEq/l, CF indicated by > 60 mEq/l

Causes of false positive sweat test

malnutrition
adrenal insufficiency
glycogen storage diseases
nephrogenic diabetes insipidus
hypothyroidism, hypoparathyroidism
G6PD
ectodermal dysplasia

Question stats

A31%
B14.8%
C24.4%
D15.6%
E14.2%

31% of users answered this question correctly

Session score = 46.9% All contents of this site are ©2009 passmedicine.com -
Terms and Conditions
passmedicine.compassmedicine.comReference ranges End session

Question 46 of 60
Which one of the following causes of pneumonia is most associated with the
development of Stevens-Johnson syndrome?ia A.ALegionellaia
B.AMycoplasmaia
C.ACoxiellaia
D.AStaphylococcusia
E.AKlebsiellaia

Stevens-Johnson syndrome
sqweqwesf erwrewfsdfs adasd dhe
Stevens-Johnson syndrome severe form of erythema multiforme associated with
mucosal involvement and systemic symptoms

Features

rash is typically maculopapular with target lesions being characteristic. May


develop into vesicles or bullae
mucosal involvement
systemic symptoms: fever, arthralgia

Causes

idiopathic
bacteria: Mycoplasma, Streptococcu s
viruses: herpes simplex virus, Orf
drugs: penicillin, sulphonamides, carbamazepine, allopurinol, NSAIDs, oral
contraceptive pill
connective tissue disease e.g. SLE

sarcoidosis
malignancy

Question stats

A5.2%
B70.7%
C4.3%
D15.7%
E4.1%

70.7% of users answered this question correctly


Session score = 45.7% All contents of this site are ©2009 passmedicine.com -
Terms and Conditions
passmedicine.compassmedicine.comReference ranges End session

Question 60 of 60
A 65-year-old life-long smoker with a significant past history of asbestos
exposure is investigated for lung cancer. Given his history of both smoking and
asbestos exposure, what is his increased risk of lung cancer?ia A.A5ia
B.A10ia
C.A50ia
D.A500ia
E.A1,000ia

Smoking and asbestos are synergistic, i.e. a smoker with asbestos exposure has a
10 * 5 = 50 times increased risk

Lung cancer: risk factors


sqweqwesf erwrewfsdfs adasd dhe
Smoking

increases risk of lung ca by a factor of 10

Other factors

asbestos - increases risk of lung ca by a factor of 5


arsenic
radon
nickel
chromate
aromatic hydrocarbon
cryptogenic fibrosing alveolitis

Factors that are NOT related

coal dust

Smoking and asbestos are synergistic, i.e. a smoker with asbestos exposure has a
10 * 5 = 50 times increased risk

Question stats

A2.7%
B8.4%
C74.5%
D11.8%
E2.6%

74.5% of users answered this question correctly

Session score = 43.3% All contents of this site are ©2009 passmedicine.com -
Terms and Conditions
passmedicine.compassmedicine.comReference ranges End session

Question 1 of 6
A 62-year-old man presents following a recent diagnosis of small cell lung
cancer. He is suffering a variety of aches and pains around his body despite
taking MST 20mg bd. He is worried that the pains could be related to the cancer
spreading to his bones. What is the most common site of bone metastases?ia
A.ASkullia
B.AMetacarpalsia
C.APelvisia
D.ARibsia
E.ASpineia

Bone metastases
sqweqwesf erwrewfsdfs adasd dhe
Most common tumour causing bone metastases (in descending order)

prostate
breast
lung

Most common site (in descending order)

spine
pelvis
ribs
skull
long bones

Question stats

A3.2%
B0.4%
C7.8%
D3.8%
E84.9%

84.9% of users answered this question correctly

Session score = 0% All contents of this site are ©2009 passmedicine.com - Terms
and Conditions

‫هللا أكبر‬...

You might also like